Sie sind auf Seite 1von 104

1.

The Parthenon was a church from 1204 until 1456, when Athens was taken by General
Mohammed the Conqueror, the Turkish sultan, who established a mosque in the building and
used the Acropolis as a fortress.
(A) who established a mosque in the building and used the Acropolis as
(B) who, establishing a mosque in the building, used the Acropolis like
(C) who, when he had established a mosque in the building, used the Acropolis like
(D) who had established a mosque in the building, using the Acropolis to be
(E) establishing a mosque in the building and using the Acropolis as
The best answer is A -- correctly supplies the past tense verbs "established" and "used" to
describe two actions performed in 1456; also, it idiomatically employs the phrase "used the
Acropolis as a fortress", in which used as means "employed in the capacity of."
B, C -- incorrect -- incorrectly replaces "as" with "like". Furthermore, in
C, "when he had established a mosque" distorts the intended meaning by stating that the first
action was completed before the second was begun.
D -- incorrect -- "had established... using" states that Mohammed had already performed the
actions before capturing Athens; In addition D includes the unidiomatic construction "using x to
be y."
E -- incorrect -- "establishing" and "using" modify Athens, thus producing an absurd statement.

2. The concept of the grand jury dates from the twelfth -century, when Henry II of England
ordered panels of common citizens should prepare lists of who were their communities'
suspected criminals.
(A) should prepare lists of who were their communities' suspected criminals
(B) would do the preparation of lists of their communities' suspected criminals
(C) preparing lists of suspected criminals in their communities
(D) the preparing of a list of suspected criminals in their communities
(E) to prepare lists of suspected criminals in their communities
Choice E is best -- the infinitive "to prepare" follows the verb
"ordered", producing the grammatical and idiomatic sequence - x ordered y to do z.
A, B -- incorrect -- "should prepare" in A and "would do" in B produce ungrammatical
sequences - x ordered y should/ would do z.
C -- incorrect -- "preparing . . . communities functions" as a participial
phrase is modifying citizens rather than as a verb phrase describing what the citizens were
ordered to do.
D -- incorrect -- the construction "ordered panels of common citizens the preparing" is
unidiomatic.
3. Chinese, the most ancient of living writing systems, consists of tens of thousands of
ideographic characters, each character miniature calligraphic composition inside its own square
frame.
(A) each character a miniature calligraphic composition inside its
(B) all the characters a miniature calligraphic ; composition inside their
(C) all the characters a miniature calligraphic composition inside its
(D) every character a miniature calligraphic composition inside their
(E) each character a miniature calligraphic composition inside their
Choice A is best -- the appositive terms "character" and "composition", both singular, agree
in number; both also agree with the singular possessive pronoun "its".
B, C, D, E -- incorrect -- three-way agreement in number stated in choice A is violated.

Labels: Idiom, Like vs as, modifier error, Subject verb agreement, Tense

Gmat Sentence Correction 36, 37

36). In contrast to large steel plants that take iron ore through all the steps needed to produce
several different kinds of steel, processing steel scrap into a specialized group of products has
enabled small mills to put capital into new technology and remain economically viable.

(A) processing steel scrap into a specialized group of products has enabled small mills to put
capital into new technology and remain

(B) processing steel scrap into a specialized group of products has enabled small mills to put
capital into new technology, remaining

(C) the processing of steel scrap into a specialized group of products has enabled small mills
to put capital into new technology, remaining

(D) small mills, by processing steel scrap into a specialized group of products, have been able
to put capital into new technology and remain

(E) small mills, by processing steel scrap into a specialized group of products, have been able
to put capital into new technology and remained

37). Any medical test will sometimes fail to detect a condition when it is present and indicate
that there is one when it is not.

(A) a condition when it is present and indicate that there is one

(B) when a condition is present and indicate that there is one

(C) a condition when it is present and indicate that it is present

(D) when a condition is present and indicate its presence

(E) the presence of a condition when it is there and indicate its presence

Answers -

36). D is the correct answer -- uses parallel verb forms to complete the construction "have
been able to put... and remain". The logical comparison here is between large steel plants and
small mills.
A, B,C -- incorrect -- illogically contrast large steel plants with [the] processing [of] steel
scrap. Further, in choices B and C remaining is
not parallel with put; consequently, it is not clear exactly what is remaining economically
viable.

E -- incorrect -- remained is not parallel with put.

37). Correct choice is C -- produces a sentence in which pronoun "it" refers clearly and
logically to the noun "condition".

A, B -- incorrect -- the phrase "indicate that there . is one" does not grammatically fit with
"when it is not" because it has no referent.

B, D -- incorrect -- are imprecise in saying that a test will fail to detect


when a condition is present, since the issue is the presence and not the timing of the condition.
Further, "its" presence in D leaves the "it" in "when it is not" without a logical referent: it
must refer to condition, not presence.

E -- incorrect -- repeats the error stated above ; also, "the presence ... when it is there" is
imprecise and redundant.

ShareThisFacebookTweetLinkedInEmailGoogle Pinterest

Posted by Prachi Pareekh at 10:29 AM No comments: Links to this post


Labels: Comparison, PARALLELISM, Pronoun error, redundancy

Wednesday, July 05, 2006


Gmat Sentence Correction 34 , 35

34). Though viewed from a distance, Saturn's main rings may appear to be smooth and
continuous, they are in fact composed of thousands of separate icy ringlets when viewed up
close.

A). Though viewed from a distance, Saturn's main rings may appear to be smooth and
continuous, they are in fact composed of thousands of separate icy ringlets when viewed up
close.

B). Though Saturn's main rings may appear smooth and continuous when viewed from a
distance, they are in fact composed of thousands of separate icy ringlets when viewed up close.

C). Saturn's main rings, when viewed from a distance, may appear to be smooth and
continuous, though when viewed up close they are in fact composed of thousands of separate
icy ringlets.
D). When viewed from a distance, Saturn's main rings may appear smooth and continuous,
but closer viewing reveals them to be composed of thousands of separate icy ringlets.

E). Though composed of thousands of separate icy ringlets if viewed up close, the main rings
of Saturn may appear smooth and continuous when they are viewed from a distance.

35). The ancient Inca city of Macchu Picchu, perched on a ridge in the Andes Mountains of
Peru, had been built at a high enough altitude that it often makes modern-day tourists from
lower elevations sick with oxygen deprivation.

A). had been built at a high enough altitude that it often makes modern-day tourists from
lower elevations sick with oxygen deprivation

B). had been built at so high of an altitude that it often makes modern-day tourists from lower
elevations sick from oxygen deprivation

C). was built at a high enough altitude that modern day tourists from lower elevations often
become sick with oxygen deprivation when visiting the city

D). was built at such a high altitude that modern-day tourists from lower elevations often
become sick from oxygen deprivation when visiting the city

E). was built at so high of an altitude that it often makes modern-day tourists from lower
elevations sick from oxygen deprivation

Answer -

34). D is the correct answer - This sentence checks the clarity of meaning and modifiers. This
choice replaces "though" with "when" and shortens "appear to be" to "appear." Further, its
use of the phrase "closer viewing reveals" clearly indicates that the close viewing only reveals
(not causes) the composition of the rings.

A - incorrect - The original sentence introduces the main clause with "though viewed from a
distance", which sets up the expectation of a contradiction that never materializes. For
example, "Though sleepy, the child stayed awake" is correct, whereas "Though sleepy, the
child may have eaten soup" is not.

Also, "when viewed up close" is placed in such a way as to illogically suggest that the rings
are composed of icy ringlets as a result of being viewed up close.

Finally, "appear to be" is redundant.

B - This choice incorrectly introduces the main clause with "though." Additionally, the
placement of "when viewed up close" illogically suggests that the rings are composed of icy
ringlets as a result of being viewed up close.
C - This choice incorrectly uses the redundant phrase "appears to be." Additionally, the use
and placement of the words "when viewed up close, they are . . ." illogically suggests that the
rings are composed of icy ringlets as a result of being viewed up close.

E - This choice incorrectly introduces the main clause with "though." Additionally, the
placement of "if viewed up close" illogically suggests that the rings are composed of icy
ringlets as a result of being viewed up close.

35). D is the correct answer - The simple past verb "was" correctly replaces the past perfect
verb "had been." The phrase "such a high altitude" replaces "high enough altitude." The
sentence is rewritten to avoid ambiguity by removing the pronoun "it." Finally, this choice
uses the phrase "sick from" in place of the unidiomatic "sick with."

A - incorrect - First, the past perfect "had been" is unnecessary here because there is only one
past event. Second, "high enough" implies that the oxygen deprivation was a goal of the
Incas. Third, "it" in this context is ambiguous: is it the altitude or the city that makes tourists
sick? Finally, "sick with" is incorrect. It should be "sick from."

B - incorrect - This choice incorrectly uses the past perfect "had been." Additionally, the
pronoun "it" is ambiguous in this context and the phrase "so high of an altitude" is
awkward.

C - This choice incorrectly uses the phrases "high enough altitude" and "sick with."

E - This choice incorrectly uses the awkward phrase "so high of an altitude." Additionally, the
pronoun "it" in this context is ambiguous.

ShareThisFacebookTweetLinkedInEmailGoogle Pinterest

Posted by Prachi Pareekh at 11:07 PM 1 comment: Links to this post


Labels: modifier error, Pronoun error, redundancy, Tense

Tuesday, July 04, 2006


Gmat Sentence Correction 32 , 33

32). . Scientists have observed large concentrations of heavy-metal deposits in the upper
twenty centimeters of Baltic Sea sediments, which are consistent with the growth of industrial
activity there.

(A) Baltic Sea sediments, which are consistent with the growth of industrial activity there

(B) Baltic Sea sediments, where the growth of industrial activity is consistent with these
findings

(C) Baltic Sea sediments, findings consistent with its growth of industrial activity
(D) sediments from the Baltic Sea, findings consistent with the growth of industrial activity in
the area

(E) sediments from the Baltic Sea, consistent with the growth of industrial activity there

33). For members of the seventeenth-century Ashanti nation in Africa, animal-hide shields
with wooden frames were essential items of military equipment, a method to protect warriors
against enemy arrows and
spears.

(A) a method to protect

(B) as a method protecting

(C) protecting

(D) as a protection of

(E) to protect

Answers -

32). D is the best choice - the phrase sediments from the Baltic Sea tells where the sediments
originate, findings provides a noun for consistent to modify, and in the area clearly identifies
where the industrial activity is growing.

A, B - incorrect - ambiguity - the words "which" and "where" appear to refer to sediments

E - incorrect - it is not clear what consistent describes.

A, C, E - incorrect - there is no logical place to which "there" or "its" could refer.

33). Choice C is best - the participle "protecting" begins a phrase that explains what the
shields did.

A , B - incorrect - awkwardly use the singular word "method" to refer to items of military
equipment rather than to the use of such items. Also, a method of protecting would be more
idiomatic than a method to protect in A or a method protecting in B.

B , D - "as" is incorrect; also, a protection in D has no noun for which it can logically
substitute.

E - is incomplete; used to protect would have been acceptable

ShareThisFacebookTweetLinkedInEmailGoogle Pinterest
Posted by Prachi Pareekh at 4:24 AM No comments: Links to this post
Labels: Pronoun error

Monday, July 03, 2006


Gmat sentence correction 30 , 31

30). Famed for his masterful use of irony, many of Guy de Maupassant's short stories have
become classics due to the author slowly revealing at the end of each piece a tragic twist of
fate.

A). Famed for his masterful use of irony, many of Guy de Maupassant's short stories have
become classics due to the author slowly revealing at the end of each piece a tragic twist of
fate.

B). Many of Guy de Maupassant's short stories have become classics because of how he
famously and masterfully uses irony, evident in the slow revelation of a tragic twist of fate at
the end of each piece.

C).Famed for using irony in a masterful way, many of Guy de Maupassant's short stories have
become classics because of the author slowly revealing a tragic twist of fate at the end of each
piece.

D). Many of Guy de Maupassant's short stories have become classics because of the author's
famed and masterful use of irony, evidenced in the slow revelation of a tragic twist of fate at
the end of each piece.

E). Many of Guy de Maupassant's short stories have become classics because he slowly
revealed a tragic twist of fate at the end of each piece, demonstrating his famed and masterful
use of irony.

31). Teachers in this country have generally been trained either to approach mathematics like
a creative activity or that they should force students to memorize rules and principles without
truly understanding how to apply them.

A). to approach mathematics like a creative activity or that they should force students to
memorize rules and principles

B). to approach mathematics like a creative activity or to force students to memorize rules and
principles

C). to approach mathematics as a creative activity or to force students to memorize rules and
principles

D). that they should approach mathematics as a creative activity or to force students to
memorize rules and principles
E). that they should approach mathematics like a creative activity or that they should force
students to memorize rules and principles

Answers -

30). D is the correct answer

A - Incorrect - sentence begins with the modifier "Famed for his masterful use of irony,"
which requires a person as its subject. However, in the sentence, "many of Guy de
Maupassant's short stories" is the subject. Moreover, the phrase "due to the author slowly
revealing" is awkward.

B - incorrect - The pronoun "he" must have a person as its antecedent, yet there is no person
in the sentence. Remember that "he" cannot refer to "Guy de Maupassant" here, since the
name is part of a possessive phrase: "Guy de Maupassant's short stories". The author himself
is not grammatically present in the sentence.

C - incorrect - The opening modifier "famed for using irony in a masterful way" incorrectly
modifies "short stories" instead of Guy de Maupassant himself. It also contains the awkward
phrase "because of the author slowly revealing."

E - incorrect - This choice incorrectly uses the pronoun "he" without a grammatical
antecedent in the sentence.

31). C is the correct choice - The construction "either X or Y" requires parallelism between X
and Y. In choice C, both X and Y are parallel infinitive phrases ("to approach . . ." and "to
force . . .").

A - incorrect - incorrectly pairs an infinitive ("to approach") with a clause ("that they
should...") in the construction "either X or Y." Moreover, the use of "like" in the phrase "to
approach mathematics like a creative activity" is incorrect. :"As" should be used instead.

B - While this choice does contain proper parallel structure, it incorrectly uses "like" instead
of "as" in the phrase "to approach mathematics like a creative activity".

D - This choice incorrectly pairs a clause ("that they should...") with an infinitive ("to
approach") in the construction "either X or Y."

E - While this choice does create a parallel construction, it awkwardly begins the parallel
elements with the words "that they" instead of the infinitive "to." Moreover, this choice
incorrectly uses "like" instead of "as" in the phrase "to approach mathematics like a creative
activity".

ShareThisFacebookTweetLinkedInEmailGoogle Pinterest
Posted by Prachi Pareekh at 6:53 AM 1 comment: Links to this post
Labels: modifier error, PARALLELISM, Pronoun error

Sunday, July 02, 2006


Gmat sentence correction 26 , 27 , 28 , 29

26). A study commissioned by the Department of Agriculture showed that if calves exercise
and associated with other calves, they will require less medication and gain weight quicker
than do those raised in confinement.

(A) associated with other calves, they will require less medication and gain weight quicker
than do

(B) associated with other calves, they require less medication and gain weight quicker than

(C) associate with other calves, they required less medication and will gain weight quicker
than do

(D) associate with other calves, they have required less medication and will gain weight more
quickly than do

(E) associate with other calves, they require less medication and gain weight more quickly
than

27). Displays of the aurora borealis, or "northern lights," can heat the atmosphere over the
arctic enough to affect the trajectories of ballistic missiles, induce electric currents that can
cause blackouts in some areas and corrosion in north-south pipelines.

(A) to affect the trajectories of ballistic missiles, induce

(B) that the trajectories of ballistic missiles are affected, induce

(C) that it affects the trajectories of ballistic missiles, induces

(D) that the trajectories of ballistic missiles are affected and induces

(E) to affect the trajectories of ballistic missiles and induce

28). The golden crab of the Gulf of Mexico has not been fished commercially in great
numbers, primarily on account of living at great depths-- 2,500 to 3,000 feet down.

(A) on account of living

(B) on account of their living


(C) because it lives

(D) because of living

(E) being they live

29). Delighted by the reported earnings for the first quarter of the fiscal year, it was decided by
the company manager to give her staff a raise.

(A) it was decided by the company manager to give her staff a raise

(B) the decision of the company manager was to give her staff a raise

(C) the company manager decided to give her staff a raise

(D) the staff was given a raise by the company manager

(E) a raise was given to the staff by the company manager

Answers -

26). Choice E, the best answer, uses the adverbial phrase "more quickly than" to modify the
verb phrase "gain weight".

In A, B, and C - "quicker than" is incorrect because an adjective should not be used to modify
a verb phrase.

A and B - incorrectly compound that present tense verb with a past tense verb, "associated".

C and D - correctly use "associate", but C follows with the past tense "required" and D with
the present perfect "have required". Both C and D incorrectly conclude with the future tense
"will gain".

27). E is the best choice.

The use of the phrasing "can heat... enough to affect" in A and E is more idiomatic than the
use of the subordinate clause beginning with "that" in B, C, and D.

B - produces an illogical and ungrammatical statement by making "induce" parallel with the
verb "heat" rather than with the appropriate form of the verb affect

C - lacks agreement in using the singular pronoun "it" to refer to the plural noun "displays"

D - is faulty because "induces" cannot fit grammatically with any noun in the sentence.
A - incorrectly separates the two infinitives "to affect" and "to induce" with a comma when it
should compound them with "and", as does E, the best choice.

28). C is the best choice.

A, B, D - the phrases "on account of" and "because of" are unidiomatic; "because", which
appears in C and E, is preferable here since "because" can introduce a complete subordinate
clause explaining the reason why the golden crab has not been fished extensively.

B , E - also produce agreement errors by using the plural pronouns "their" and "they" to
refer to the singular noun "crab".

D - like A, fails to provide a noun or pronoun to perform the action of living.

C - which uses "because" and "it" as the singular subject of a clause, is the best choice.

29). C is the best answer. Grammatically, the participial phrase beginning delighted must
modify the subject of the main clause. Because it is the manager who was delighted, choice C,
in which the company manager appears as the subject, is the best answer.

A, B, D, E - create illogical statements by using it, the decision, the staff, and a
raise,respectively, as the sentence subject.

A , D , E - Use of the passive voice

ShareThisFacebookTweetLinkedInEmailGoogle Pinterest

Posted by Prachi Pareekh at 9:34 PM 1 comment: Links to this post


Labels: modifier error, Passive, Pronoun error, Subject verb agreement, Tense

Friday, June 30, 2006


Gmat sentence correction 24 , 25

Gmat Sentence Correction 24 & 25

24). Based on accounts of various ancient writers, scholars have painted a sketchy picture of
the activities of an all-female cult that, perhaps as early as the sixth century B.C., worshipped
a goddess known in Latin as Bona Dea, "the good goddess."

(A) Based on accounts of various ancient writers

(B) Basing it on various ancient writers' accounts

(C) With accounts of various ancient writers used for a basis


(D) By the accounts of various ancient writers they used

(E) Using accounts of various ancient writers

25). The number of acres destroyed by wildfires, which have become an ongoing threat due to
drought and booming population density, have increased dramatically over the past several
years, prompting major concern among local politicians.

A). which have become an ongoing threat due to drought and booming population density,
have increased

B). which have become an ongoing threat due to drought and booming population density,
have been increasing

C). which has become an ongoing threat because of drought and booming population density,
has increased

D). which have become an ongoing threat due to drought and booming population density,
has increased

E). which have become an ongoing threat because of drought and booming population
density, has increased

Answers-

24). E is the best choice.

A - Dangling modifier - the introductory clause beginning Based on modifies scholars, the
noun that immediately follows it.
In other words, A says that scholars were based on the accounts of various ancient writers.

B - is awkward and imprecise - referent of pronoun "it" is not clear.

C, D - are also wordy and awkward.


Further in D - By the accounts... they used is an unidiomatic and roundabout way of saying
that scholars used me accounts.

E - is clear and concise - correctly uses a present participle (or "-ing" verb) to introduce the
modifier describing how the scholars worked.

25). E is the correct choice.

A - The subject of the original sentence is "the number of acres", which is singular. The main
verb, however, is "have increased", which is plural. We need to find a choice that replaces
"have" with "has". Moreover, the phrase "due to" is incorrect in this context. "Due to" is a
phrase that must describe a noun. "The fire was due to drought" is correct, but "There was a
fire due to drought" is not. When describing a verb phrase, "because of" is preferable:
"There was a fire because of drought."

B - The plural verb "have been" does not agree with the singular subject "the number of
acres." Additionally, the passive voice "have been increased" is incorrect. Finally, the phrase
"due to drought . . ." is unidiomatic since "because of" (not "due to") should be used to
modify the verb phrase "have become an ongoing threat."

C - The modifier "which has become an ongoing threat . . ." contains the singular verb "has"
which does not agree with the plural subject "wildfires."

D - The phrase "due to drought . . ." is unidiomatic since "because of" (not "due to") should
be used to modify the verb phrase "have become an ongoing threat."

E - Correct - The singular verb "has" agrees with the singular subject "the number of acres."
Additionally, "because of drought . . ." is properly used to modify the verb phrase "have
become an ongoing threat."

ShareThisFacebookTweetLinkedInEmailGoogle Pinterest

Posted by Prachi Pareekh at 12:11 AM 1 comment: Links to this post


Labels: Idiom, modifier error, Pronoun error, Subject verb agreement, wordy

Wednesday, June 28, 2006


Gmat sentence correction 22 , 23

22). A recent study has found that within the past few years, many doctors had elected early
retirement rather than face the threats of lawsuits and the rising costs of malpractice
insurance.

(A) had elected early retirement rather than face

(B) had elected early retirement instead of facing

(C) have elected retiring early instead of facing

(D) have elected to retire early rather than facing

(E) have elected to retire early rather than face

23). In metalwork one advantage of adhesive-bonding over spot-welding is that the contact,
and hence the bonding, is effected continuously over a broad surface instead of a series of
regularly spaced points with no bonding in between.
A) instead of

B) as opposed to

C) in contrast with

D) rather than at

E) as against being at

Answer -

22). Best answer choice is E

A, B - wrong tense 'had elected'


C - here preference is shown so use of 'instead of' is incorrect.
D - violating parallelism - D has parallelism problem. ...elected to retire....rather than
facing....

Hence E is the best choice.

23). D is the best choice, as preference is suggested in the sentence.

A - "instead of " - incorrect - not a substitute


B - "as opposed to" - incorrect - there is no argument
C - "in contrast with" - incorrect
E - "being" - incorrect - as use of 'being'- not entertained in Gmat .

ShareThisFacebookTweetLinkedInEmailGoogle Pinterest

Posted by Prachi Pareekh at 11:08 PM No comments: Links to this post


Labels: PARALLELISM, Rather than vs instead of, Tense

Gmat sentence correction 21

21). Patrice, whom according to the classical musician community is a virtuoso, plays in a
unique style which is all her own but which also embodies a warmth prevalent during the
golden age of violin playing.

A). whom according to the classical musician community is a virtuoso, plays in a unique style
which is all her own but which also embodies

B). considered a virtuoso by the community of classical musicians, plays in a style all her own
which at the same time embodies
C). regarded by classical musicians as being a virtuoso, plays in a unique style all her own yet
embodies

D). regarded by classical musicians as a virtuoso, who plays in a style all her own which at the
same time embodies

E). whom the community of classical musicians would consider to be a virtuoso, plays in a
unique style while at the same time embodying

Answer - B is the best choice.Click on the link below to see the explanation.

Explanation

ShareThisFacebookTweetLinkedInEmailGoogle Pinterest

Posted by Prachi Pareekh at 3:56 AM No comments: Links to this post

Friday, June 23, 2006


Gmat sentence correction 19 20

19). If anyone at InterCom Financial Advisers would have anticipated, or even suspected, the
impending sale of the Koniko kelp processing plant, they would have advised owners of
Koniko stock to unload all shares immediately.

(A) If anyone at InterCom Financial Advisers would have anticipated

(B) Had anyone at InterCom Financial Advisers anticipated

(C) If any people at InterCom Financial Advisers would have anticipated

(D) If any people at InterCom Financial Advisers had anticipated

(E) If anybody at InterCom Financial Advisers anticipated

20). If the reporter would have known the landlord's side of the story, she would not have
written an article so favorable to the 81-year-old tenant.

(A) would have known the landlord's side of the story, she would not have written

(B) would of known the landlord's side of the story, she would not of written

(C) had known the landlord's side of the story, she would not have written
(D) had known the landlord's side of the story, she would not have wrote

(E) knew the landlord's side of the story, she would not have written

Answers -

19). D is the right answer - Correct tense


anyone - singular
any people - plural,
use of " they would .. " in later half of the sentence implies we need plural .

20). C is the right choice.

Whenever you come across if ... then tense construction in a sentence,

IF Clause Then Clause

PAST PERFECT ---- WOULD/COULD + HAVE + PARTICIPLE

If Ram had won... he would have given.....

ShareThisFacebookTweetLinkedInEmailGoogle Pinterest

Posted by Prachi Pareekh at 3:55 AM No comments: Links to this post


Labels: If then construction, Tense

Tuesday, June 20, 2006


Gmat Sentence Correction 17 , 18

Gmat Sentence Correction 17 & 18

17). Charlotte Perkins Gilman, a late nineteenth-century feminist, called for urban apartment
houses including child-care facilities and clustered suburban houses including communal
eating and social facilities.

(A) including child-care facilities and clustered suburban houses including communal eating
and social facilities

(B) that included child-care facilities, and for clustered suburban houses to include
communal eating and social facilities

(C) with child-care facilities included and for clustered suburban houses to include communal
eating and social facilities
(D) that included child-care facilities and for clustered suburban houses with communal
eating and social facilities

(E) to include child-care facilities and for clustered suburban houses with communal eating
and social facilities included

18). No one but him could have told them that the king was I.

(A) him could have told them that the king was I.

(B) he could have told them that the king was me.

(C) him could have told them the king was me.

(D) he could have told them that the king was I.

(E) him could had told them that the king was I.

Answer -

17). D is the right choice.

A - AMBIGUITY - including..... - not clear - it is hard to tell whether Gilman called for urban
apartment houses that included child-care facilities or whether such facilities represent one
variety of the urban apartment houses she wanted to built.

C,E - incorrect - preposition + noun + participle, Gmat almost never entertains this pattern --
communal eating and social facilities included.

B - to include - unidiomatic

18). A is the answer.Gmat grammar tip --

"except", "no one but" should be followed by object form of Pronoun.

No one but -- in other words means - except.

Always use subject form of Pronoun after the linking verb.--(Here linking verb 'was' so we
use subject form of pronoun i.e I)

ShareThisFacebookTweetLinkedInEmailGoogle Pinterest
Posted by Prachi Pareekh at 10:52 AM 4 comments: Links to this post
Labels: Pronoun error

Gmat Sentence Correction 15 , 16

Gmat Sentence Correction 15 & 16

15). The suspect is facing charges that he illegally published telephone credit card numbers,
intending that they be used by others to avoid telephone billings.

A. intending that they be used by others to avoid telephone billings.

B. with the intent of others using them to avoid telephone billings.

C. intending that telephone billings could be avoided by others using them.

D. with the intent of their use by others to avoid telephone billings.

E. with the intent that telephone billings would be avoided by others through their use.

16). Teratomas are unusual forms of cancer because they are composed of tissues such as
tooth and bone not normally found in the organ in which the tumor appears.

A. because they are composed of tissues such as tooth and bone

B. because they are composed of tissues like tooth and bone that are

C. because they are composed of tissues, like tooth and bone, tissues

D. in that their composition , tissues such as tooth and bone, is

E. in that they are composed of tissues such as tooth and bone, tissues

Answer -

15). A is the best choice - The correct idioms are -


1. Intent on verb+ing.
2. intend to verb -- applying 2nd, A is the best choice.

B, D, E -- "numbers" cannot have intent.

C -- use "shall " or "should" instead of "could". Further, use of "them" is ambiguous as it
can either refer to "numbers" or to "billings".

16). E is the best choice.It is a modifier question.(To know about modifiers click on links
below).
A - "not normally found..." modifies "bone"
B - "that.." modifies "bone" again
D - Composition = tissues ? One is singular, the other is plural.

In gmat "like" is used to show similarity whereas "such as" means example. By this
reasoning we straight away eliminate choices B, C.

Further "because" is used to express a simple relationship whereas "in that" qualifies the
previous statement. In this sentence use of because is wrong, also in Gmat "in that " is
preferred over "because".

http://gmat-grammar.blogspot.com

ShareThisFacebookTweetLinkedInEmailGoogle Pinterest

Posted by Prachi Pareekh at 2:43 AM No comments: Links to this post


Labels: Idiom, Like vs such as, modifier error, Pronoun error

Monday, June 19, 2006


Gmat Sentence Correction 13, 14

SENTENCE CORRECTION 13 & 14

13). In the Renaissance, painters were so impressed with Leonardo da Vinci that they ignored
their own training and designate as a masterpiece anything he painted.

(A) were so impressed with Leonardo da Vinci that they ignored

(B) were impressed with Leonardo da Vinci to such an extent that they were to ignore

(C) were so impressed with Leonardo da Vinci as to ignore

(D) were so impressed with Leonardo da Vinci that they had to ignore

(E) were as impressed with Leonardo da Vinci as to ignore

14). Many of them chiseled from solid rock centuries ago, the mountainous regions of
northern Ethiopia are dotted with hundreds of monesteries.

(A). Many of them chiseled from solid rock centuries ago, the mountainous regions of
northern Ethiopia are dotted with hundreds of monesteries.

(B). chiseled from solid rock centuries ago,the mountainous regions of northern Ethiopia are
dotted with hundreds of monesteries.
(C). hundreds of monesteries, many of them chiseled from solid rock centuries ago, re dotting
the mountainous regions of northern Ethiopia.

(D). The mountainous regions of northern Ethiopia are dotted with hundreds of monesteries,
many of which are chiseled from solid rock centuries ago.

(E). The mountainous regions of northern Ethiopia are dotted with hundreds of
monesteries,many of them chiseled from solid rock centuries ago.

Answers :-

13). The correct answer is C.

A - wrong - they ignored..... and designate ... this is incorrect... either use present tense or
past.. not both..

We use 'so as' to mean with the purpose or result - must be followed by an infinitive. This rule
this rule applies in the given sentence.
Painters were impressed... [as a result] they ignored and designated...

The last part of the sentence uses 'designate'. It's a hint that 'ignore' should be used in its
infinitive form as well (because of the rule and to preserve parallelism).

Note - GMAT generally prefers "so that" when applicable. However, because there is an
infinitive form "designate" later in the sentence, you have to use "so as to" to maintain
parallelism in verb tense.

14). E is the best choice.

A,B - Wrong because the chiseled ones are monastries and not the mountain regions.

C - Run-on sentence. It must be many of which.

D - Use of 'are' is incorrect.

E - many of .....ago - absolute phrase - so this is right choice

ShareThisFacebookTweetLinkedInEmailGoogle Pinterest

Posted by Prachi Pareekh at 3:00 AM No comments: Links to this post


Labels: Tense
Tuesday, June 13, 2006
Gmat Sentence Correction 11, 12

GMAT SENTENCE CORRECTION -- 11 & 12

11). Three hundred guests is as many as even the most ambitious host or
hostess should invite, and this number of guests is advisable only when
circumstances actually demand it.

A) as many as even

B) so many as even

C) so many that even

D) as many that even

E) even so many that

12). The ideal citeis of Ebenezer Howard, Frank Lloyd Wright, and Le Corbusier, different
from one another as they were, were all based on a belief that physical order in itself could
bring about the good life.

A). different from one another as they were

B). different one from the other as they were

C). different from each other as every one was

D). differing as each was, one from the other

E). although each differed from

Answer

11). A --- 2 idioms involved here are ---

1- As many as - to the same extent, degree (equal to) -- Ram was as sad as someone could ever
be

2- So... that..- To such an extent/degree that...usually leads to a conclusion -- Ram was so sad
that he decided to leave his job.
In the question :-- Three hundred guests is as many as even the most ambitious host or
hostess should invite.

As can be seen, "even the most ambitious host or hostess should invite" is not the conclusion
of the 300 guests.

Instead,"as many as" will make the proper link between the two parts => that the 300 guests
is equal to what the most ambitious host should invite.

12). A "as" here is somewhat similar to "even though".


Other catch in this sentence lies in the fact that we often omit the first "as" when we invert,
here it implies -- "as different from one another as they were" is same as "different from one
another as they were".

ShareThisFacebookTweetLinkedInEmailGoogle Pinterest

Posted by Prachi Pareekh at 9:55 AM No comments: Links to this post


Labels: Idiom

Monday, June 12, 2006


GMAT Sentence Correction 1-10

I will be posting few sentence correction questions in a day daily, the


explanation and correct answer to each question will be posted by me at
the end of each day, hope this would help all those who are preparing for
Gmat.

1. A calendar stick carved centuries ago by the Winnebago tribe may


provide the first evidence that the North American Indians have developed
advanced full-year calendars basing them on systematic astronomical
observation.

(A) that the North American Indians have developed advanced full-year
calendars basing them
(B) of the North American Indians who have developed advanced full-year
calendars and based them
(C) of the development of advanced full-year calendars by North American Indians, basing
them
(D) of the North American Indians and their development of advanced full-year calendars
based
(E) that the North American Indians developed advanced full-year calendars based

2. A 1972 agreement between Canada and the United States reduced the amount of
phosphates that municipalities had been allowed to dump into the Great Lakes.

(A) reduced the amount of phosphates that municipalities had been allowed to dump
(B) reduced the phosphate amount that municipalities had been dumping
(C) reduces the phosphate amount municipalities have been allowed to dump
(D) reduced the amount of phosphates that municipalities are allowed to dump
(E) reduces the amount of phosphates allowed for dumping by municipalities

3. A collection of 38 poems by Phillis Wheatley, a slave, was published in the 1770s, the first
book by a Black woman and it was only the second published by an American woman.

(A) it was only the second published by an American woman


(B) it was only the second that an American woman published
(C) the second one only published by an American woman
(D) the second one only that an American woman published
(E) only the second published by an American woman

4. A common disability in test pilots is hearing impairment, a consequence of sitting too close
to large jet engines for long periods of time.

(A) a consequence of sitting too close to large jet engines for long periods
of time
(B) a consequence from sitting for long periods of time too near to large jet
engines
(C) a consequence which resulted from sitting too close to large jet engines
for long periods of time
(D) damaged from sitting too near to large jet engines for long periods of
time
(E) damaged because they sat too close to large jet engines for long
periods of time

5. A controversial figure throughout most of his public life, the Black


leader Marcus Garvey advocated that some Blacks return to Africa, the
land that, to him, symbolized the possibility of freedom.

(A) that some Blacks return to Africa, the land that, to him, symbolized the
possibility of freedom
(B) that some Blacks return to the African land symbolizing the possibility
of freedom to him
(C) that some Blacks return to Africa which was the land which
symbolized the possibility of freedom to him
(D) some Blacks returning to Africa which was the land that to him
symbolized the possibility of freedom
(E) some Blacks return to the land symbolizing the possibility of freedom
to him, Africa

6. A fire in an enclosed space burns with the aid of reflected radiation that
preheats the fuel, making ignition much easier and flames spreading more
quickly.

(A) flames spreading


(B) flame spreads
(C) flames are caused to spread
(D) causing flames to spread
(E) causing spreading of the flames

7. A firm that specializes in the analysis of handwriting claims from a one-


page writing sample that it can assess more than three hundred
personality traits, including enthusiasm, imagination, and ambition.

(A) from a one-page writing sample that it can assess


(B) from a one-page writing sample it has the ability of assessing
(C) the ability, from a one-page writing sample, of assessing
(D) to be able, from a one-page writing sample, to assess
(E) being able to assess, from a one-page writing sample,

8. A huge flying reptile that died out with the dinosaurs some 65 million
years ago, the Quetzalcoatlus had a wingspan of 36 feet, believed to be the
largest flying creature the world has ever seen.

(A) believed to be
(B) and that is believed to be
(C) and it is believed to have been
(D) which was, it is believed,
(E) which is believed to be

9. A Labor Department study states that the numbers of women employed


outside the home grew by more than a thirty-five percent increase in the
past decade and accounted for more than sixty-two percent of the total
growth in the civilian work force.

(A) numbers of women employed outside the home grew by more than a
thirty-five percent increase
(B) numbers of women employed outside the home grew more than thirty-
five percent
(C) numbers of women employed outside the home were raised by more
than thirty-five percent
(D) number of women employed outside the home increased by more than
thirty-five percent
(E) number of women employed outside the home was raised by more
than a thirty-five percent increase

10. A large rise in the number of housing starts in the coming year should
boost new construction dollars by several billion dollars, making the
construction industrys economic health much more robust than five
years ago.

(A) making the construction industrys economic health much more


robust than five years ago
(B) and make the construction industrys economic health much more
robust than five years ago
(C) making the construction industrys economic health much more robust
than it was five years ago
(D) to make the construction industrys economic health much more
robust than five years ago
(E) in making the construction industrys economic health much more
robust than it as five years ago

Answers

1.) E is the best choice as it only maintains parallelism -- carved... developed....based

2.) OA - D.
Since the dumping continued after the date of the agreement, the past perfect verb had been
allowed in choice A is incorrect. The verb should instead be the present are allowed(if the
agreement remained in effect when the sentence was written) or the past were allowed(if the
agreement was no longer in effect when the sentence was written). B changes the meaning of
the sentence by stating that the amount of phosphate that municipalities were dumping
whereas stem only talks about the limit and not whether they were up to the limit.C, E use
wrong tense reduces, should be the past tense reduced. Further allowed for dumping in E is
unidiomatic. Also the phrase amount of phosphate is clear and idiomatically correct, whereas
phosphate amount is not idiomatic

3). The best choice is E, Here parallelism has to be maintained between two nouns and not
two sentences, thus options A and B are ruled out.Also in choices A and B use of it denotes
redundancy. This leaves us with C, D, and E. Choices C and D are wordy, also according to
the grammar rule one should put only as close as possible to the word it is suppose to
modify.Further choice D is depicting incorrect placing of active and passive voice
constructions in the sentence.

4). A is the best choice.

5). A is the best answer. D, E are ruled out because of use of apostrophe which is changing the
expression to noun form. Option B is unclear and suffers from ambiguity and in option C the
referent of which is not clear.

6). D is the best choice.

7). D is the best answer. It correctly uses an infinitive to connect the verb "claims" with the
firm's assertion : claims to be able .....to assess....All of the other choices use ungrammatical
or unclear constructions after claims. Choices A and B present clauses that should be
introduced by "claims that". In A, placing that after sample rather than after claims produces
the unintended statement that the claim itself is made on the basis of a single one - page
writing sample.Also in B, the ability of assessing is unidiomatic.Choice C repeats this second
fault and uses the unidiomatic claims the ability.Choice E uses the ungrammatical claims
being able to assess.

8). C is the best choice.


A - incorrect - participial phrase is incorrectly referring to 36 feet. Further wrongly suggests
that Quetzalcoatlus still exists though the real meaning conveyed here is that it is still
considered the largest flying creature (present perfect)
B - incorrect - "that" is modifying "a wingspan", instead of modifying "Quetzalcoatlus".
D, E - incorrect - "which" modifying "a wingspan", instead of modifying "Quetzalcoatlus"

9). D is the best choice.Because a count of women employed outside the home at any given
time will be expressed by a single number, the use of the plural noun numbers in choices A, B,
and C is illogical. In A, the phrase grew by more than a thirty-five percent increase is
redundant and wordy, since the sense of increase is implicit in the verb grew. In C and E, the
passive verb forms were raised and was raised are inappropriate because there is no
identifiable agent responsible for the raising of the number of women employed. In choice E,
was raised by ... increase is redundant. Choice D, which presents the comparison logically and
idiomatically, is the best answer.

10). C is the best answer.Wrong comparison - health to five years ago eliminate option A, B,
and D .In option E in making is not necessary, thus making it unnecessarily wordy.

79). Federal incentives now encourage investing capital in commercial office buildings despite
vacancy rates in existing structures that are exceptionally high and no demand for new
construction.

(A) investing capital in commercial office buildings despite vacancy rates in existing
structures that are exceptionally high and

(B) capital investment in commercial office buildings, even though vacancy rates in existing
structures are exceptionally high and there is

(C) capital to be invested in commercial office buildings even though there are exceptionally
high vacancy rates in existing structures with

(D) investing capital in commercial office buildings even though the vacancy rates are
exceptionally high in existing structures with

(E) capital investment in commercial office buildings despite vacancy rates in existing
structures that are exceptionally high, and although there is

80). A large and increasingly influential sector of publishing, 20 percent of all the National
and American Book awards since 1950 have gone to university-press books.

(A) A large and increasingly influential sector of publishing, 20 percent of all the National
and American Book awards since 1950 have gone to university-press books.

(B) A large and increasingly influential sector of publishing, university-press books have won
20 percent of all the National and American Book awards since 1950.

(C) Increasingly influential as a large sector of publishing, 20 percent of all the National and
American Book awards since 1950 have gone to university-press books.

(D) Since 1950, a large and increasingly influential sector of publishing, 20 percent of all the
National and American Book awards have gone to university-press books.

(E) Since 1950, university-press books, a large and increasingly influential sector of
publishing, won 20 percent of all the National and American Book awards from then on.

Answers --

79). Best choice is B - Federal incentives now encourage (capital investment) (noun) in
commercial office buildings, even though vacancy rates in existing structures are
exceptionally high and there is no demand for new construction(maintains parallelism)
A, E - incorrect - suggest structures are high and not the vacancy rates C -awkward - passive
voice
D - incorrect - it should be and instead of with, further word that follows encourage must be a
noun and not a verb.

80). B is the best choice


A, C, D - incorrect - dangling modifier ( Book awards is modifying sector of publishing)
E - incorrect - Wrong tense..it should be present perfect tense -1950 --- till date.
Also from then on is redundant.

ShareThisFacebookTweetLinkedInEmailGoogle Pinterest

Posted by Prachi Pareekh at 10:24 PM 6 comments: Links to this post


Labels: modifier error, PARALLELISM, Passive, redundancy, Tense

Gmat Sentence Correction 77, 78

77). Oberlin College in Ohio was a renegade institution at its 1833 founding for deciding to
accept both men and women as students.

(A) at its 1833 founding for deciding to accept

(B) for the decision at its 1833 founding to accept


(C) when it was founded in 1833 for its decision to accept

(D) in deciding at its founding in 1833 to accept

(E) by deciding at its founding in 1833 on the acceptance of

78). In feudal Europe, urban areas developed from clusters of houses where peasants lived
and commuted to farmlands in the countryside, unlike homesteading policies in the American
West that required residency on the land itself in order to obtain eventual ownership.

(A) In feudal Europe, urban areas developed from clusters of houses where peasants lived and
commuted to farmlands in the countryside, unlike homesteading policies in the American West
that

(B) In feudal Europe, urban areas developed from clusters of houses where peasants lived and
from which they commuted to farmlands in the countryside, but in the American West
homesteading policies

(C) Unlike feudal Europe where urban areas developed from clusters of houses where
peasants lived and commuted to farmlands in the countryside, the American West's
homesteading policies

(D) Unlike feudal Europe where urban areas developed from clusters of houses where
peasants lived and commuted to farmlands in the countryside, the homesteading policies of
the American West

(E) Urban areas developed from clusters of houses where peasants lived from which they
commuted to farmlands in the countryside in feudal Europe, unlike the American West where
homesteading policies

Answers --

77). Best choice is D


A -- incorrect -- implies - as if Oberlin College was a renegade institution specifically at its
founding which was for a very short duration.
B -- incorrect -- implies - as if there were more than one founding.
C -- incorrect -- implies - as if it was founded for the decision to accept men and women
E -- incorrect -- wordy - "on the acceptance of."

78). The best choice is B -- Correctly comparing fuedal Europe to American West
A, C, D -- incorrect -- comparing Europe to policies.
E -- incorrect -- awkward.

ShareThisFacebookTweetLinkedInEmailGoogle Pinterest
Posted by Prachi Pareekh at 1:33 AM No comments: Links to this post
Labels: Comparison, wordy

Gmat Sentence Correction 75, 76

75). It is an oversimplified view of cattle raising to say that all one has to do with cattle is leave
them alone while they feed themselves, corral them and to drive them to market when the time
is ripe.

(A) all one has to do with cattle is leave them alone while they feed themselves, corral them,
and to

(B) all one has to do with cattle is to leave them alone to feed themselves, to corral them, and

(C) all one has to do with cattle is leave them alone while they feed themselves and then corral
them and

(D) the only thing that has to be done with cattle is leave them alone while they feed
themselves, corral them, and

(E) the only thing that has to be done with cattle is to leave them alone while they feed
themselves, to corral them, and

76). Although dozens of New York's small museums are either devoted to local history of
various ethnic groups, there are many one-of-a-kind museums from Manhattan to the Bronx
that are open for exploration on summer weekends.

(A) Although dozens of New York's small museums are either devoted to local history or
various ethnic groups, there are

(B) Although dozens of New York's small museums are devoted to local history or various
ethnic groups,

(C) Dozens of New York's small museums are devoted to local history or various ethnic
groups, but there are

(D) Dozens of New York's small museums are devoted to local history or various ethnic
groups, and there are also

(E) Devoted to local history or various ethnic groups, dozens of New York's small museums
and also

Answers --

75). C is the best answer .


A, B -- incorrect -- parallelism "to leave them, to corral them and to drive them" would be the
correct form.
D, E -- incorrect -- use of "the only thing" is wrong as talking about more than one action.

76). Best answer is D.


A, B -- incorrect -- use of although incorrect, it is used to show contrast, but there is no
contrast here.
C -- incorrect -- use of "but" wrong as sentence does not have a contrast
E -- incorrect -- awkward

ShareThisFacebookTweetLinkedInEmailGoogle Pinterest

Posted by Prachi Pareekh at 12:33 AM No comments: Links to this post


Labels: PARALLELISM

Sunday, October 08, 2006


Gmat Sentence Correction 73, 74

73). Building large new hospitals in the bistate area would constitute a wasteful use of
resources, on the basis of avoidance of duplicated facilities alone.

(A) on the basis of avoidance of duplicated facilities alone

(B) on the grounds of avoiding duplicated facilities alone

(C) solely in that duplicated facilities should be avoided

(D) while the duplication of facilities should be avoided

(E) if only because the duplication of facilities should be avoided

74). Even though its per capita food supply hardly increased during two decades, stringent
rationing and planned distribution have allowed the Peoples Republic of China to ensure
nutritional levels of 2,000 calories per person per day for its population.

(A) Even though its per capita food supply hardly increased during

(B) Even though its per capita food supply has hardly increased in

(C) Despite its per capita food supply hardly increasing over

(D) Despite there being hardly any increase in its per capita food supply during

(E) Although there is hardly any increase in per capita food supply for
Answers --

73). Passive constructions are not always wrong on GMAT


E - correct - if only because is correct idiomatic usage -- the main clause needs to be followed
logically by because hence if only because is absolutely correct here.
A - incorrect - wordy and illogical
B,C - incorrect - duplicated facilities .... duplication should be avoided and not the facilities...
C - incorrect - in that unidiomatic
D - incorrect - Usage of while not right

74). B is the best answer.


OE - In choice A, the simple past tense hardly increased does not match the present perfect
have allowed; consequently, it seems that two different time periods are being discussed. In B,
the best choice, has hardly increased parallels have allowed to indicate that the events
described took place at the same time. Also in is the best word here for making a comparison
between the beginning and the end of the twenty-year period. Choices C and D are awkward
and unidiomatic, and choice E fails to specify where there was no increase in per capita food
supply.

ShareThisFacebookTweetLinkedInEmailGoogle Pinterest

Posted by Prachi Pareekh at 9:26 PM No comments: Links to this post


Labels: Idiom, Passive, Tense

Tuesday, September 19, 2006


Gmat Sentence Correction 70, 71, 72

70). Unlike a typical automobile loan, which requires a fifteen- to twenty-percent down
payment, the lease-loan buyer is not required to make an initial deposit on the new vehicle.

(A). the lease-loan buyer is not required to make

(B). with lease-loan buying there is no requirement of

(C). lease-loan buyers are not required to make

(D). for the lease-loan buyer there is no requirement of

(E). a lease-loan does not require the buyer to make

71). . In his research paper, Dr. Frosh, medical director of the Payne Whitney Clinic,
distinguishes mood swings. which may be violent without their being grounded in mental
disease, from genuine manic-depressive psychosis.
(A). mood swings, which may be violent without their being grounded in mental disease, from
genuine manic-depressive psychosis

(B). mood swings, perhaps violent without being grounded in mental disease, and genuine
manic-depressive psychosis ,

(C). between mood swings, which may be violent without being grounded in mental disease,
and genuine manic-depressive psychosis

(D). between mood swings, perhaps violent without being grounded in mental disease, from
genuine manic-depressive psychosis

(E). genuine manic-depressive psychosis and mood swings, which may be violent without
being grounded in mental disease

72). Paleontologists believe that fragments of a primate jawbone unearthed in Burma and
estimated at 40 to 44 million years old provide evidence of a crucial step along the
evolutionary path that led to human beings.

(A). at 40 to 44 million years old provide evidence of

(B). as being 40 to 44 million years old provides evidence of

(C). that it is 40 to 44 million years old provides evidence of what was

(D). to be 40 to 44 million years old provide evidence of

(E). as 40 to 44 million years old provides evidence of what was

Answers -

70). Choice E, the best answer - correctly uses a parallel construction to draw a logical
comparison: Unlike a typical automobile loan,... a lease-loan....

A - incorrect - illogically compares an automobile loan, an inanimate thing, with a lease-loan


buyer, a person.

C - incorrect - buyers makes the comparison inconsistent in number as well as illogical.

B, D - incorrect - are syntactically and logically flawed because each attempts to compare the
noun loan and a prepositional phrase: with lease-loan buying in B and/or the lease-loan
buyer in D. These choices are also imprecise and awkward.

71). The best choice is C - uses the idiomatically correct expression distinguishes between x
and y and because it provides a structure in which the relative clause beginning "which may
be violent" clearly modifies "mood swings".
A, B, D, E - incorrect - use distinguishes in unidiomatic constructions. Additionally, their in A
is intrusive and unnecessary, and the modifier of mood swings in B and D (perhaps violent) is
awkward and less clear than the more developed clause which may be violent.

72). D, the best choice - correctly follows estimated with to be. The other choices present
structures that are not idiomatic when used in conjunction with estimated.

A - incorrect - unidiomatic

B, C, E - incorrect - all mismatch the singular verb "provides" with its plural subject
"fragments" .Further in C and E, what was is unnecessary and wordy. In choice C, the use of
the verb phrase estimated that it is produces an ungrammatical sentence.

ShareThisFacebookTweetLinkedInEmailGoogle Pinterest

Posted by Prachi Pareekh at 12:34 AM No comments: Links to this post


Labels: Comparison, Idiom, PARALLELISM, Subject verb agreement

Gmat Sentence Correction 67, 68, 69

67). Carbon-14 dating reveals that the megalithic monuments in Brittany are nearly 2, 000
years as old as any of their supposed Mediterranean predecessors.

(A) as old as any of their supposed

(B) older than any of their supposed

(C) as old as their supposed

(D) older than any of their supposedly

(E) as old as their supposedly

68). The only way for growers to salvage frozen citrus is to process them quickly into juice
concentrate before they rot when warmer weather returns.

(A) to process them quickly into juice concentrate before they rot when warmer weather
returns

(B) if they are quickly processed into juice concentrate before warmer weather returns to rot
them

(C) for them to be processed quickly into juice concentrate before the fruit rots when warmer
weather returns
(D) if the fruit is quickly processed into juice concentrate before they rot when warmer
weather returns

(E) to have it quickly processed into juice concentrate before warmer weather returns and rots
the fruit

69). Some bat caves, like honeybee hives, have residents that take on different duties such as
defending the entrance, acting as sentinels and to sound a warning at the approach of danger,
and scouting outside the cave for new food and roosting sites.

(A) acting as sentinels and to sound

(B) acting as sentinels and sounding

(C) to act as sentinels and sound

(D) to act as sentinels and to sound

(E) to act as a sentinel sounding

Answer -

67). B is the best choice older than makes the point of comparison clear. B also uses the
adjective "supposed" correctly.

A, C, E incorrect Illogical comparison - The expression as old as indicates equality of age,


but the sentence indicates that the monuments by Brittany monuments predate the
Mediterranean 2,000 years

D, E incorrect adverb supposedly used to modify the noun phrase Mediterranean


predecessors.

68). The best choice is E - has parallel infinitives and uses to to refer unambiguously to
citrus fruit. The cause-and-effect also expresses the relationship between the return of warmer
weather and the rotting of the fruit. For parallelism, the linking verb is should link two
infinitives : The only way to salvage. . . is to process.

A incorrect - the plural pronouns them and they do not agree with the singular noun
citrus.

B, C, D incorrect - do not begin with an infinitive. and all present pronoun errors : the
plural pronouns them and they grammatically refer to citrus or fruit and doesnot/cannot
refer to farmers.

69). B is the best choice - Because the verb phrases used to describe the bats' duties are
governed by the phrase different duties such as, they should each be expressed in the present
participial (or "-ing") form to parallel defending and scouting. Only B uses the correct
idiom, and observes parallelism.

A, C, D, E - incorrect - all violate parallelism by employing infinitives (to. . . ) in place of


participial destinations.

ShareThisFacebookTweetLinkedInEmailGoogle Pinterest

Posted by Prachi Pareekh at 12:23 AM No comments: Links to this post


Labels: Comparison, modifier error, PARALLELISM, Pronoun error

Gmat Sentence Correction 65, 66

65). The Wallerstein study indicates that even after a decade young men and women still
experience some of the effects of a divorce occurring when a child.

(A) occurring when a child

(B) occurring when children

(C) that occurred when a child

(D) that occurred when they were children

(E) that has occurred as each was a child

66). Since 1981, when the farm Journal began, the number of acres overseen by professional
farm-management companies have grown from 48 million to nearly 59 million. an area that is
about Colorado 's size.

(A) have grown from 48 million to nearly 59 million, an area that is about Colorado 's size

(B) have grown from 48 million to nearly 59 million, about the size of Colorado

(C) has grown from 48 million to nearly 59 million, an area about the size of Colorado

(D) has grown from 48 million up to nearly 59 million, an area about the size of Colorado 's

(E) has grown from 48 million up to nearly 59 million, about Colorado 's size

Answers -

65). Choice D is best. The phrasing a divorce that occurred when they were children
correctly uses the relative clause that occurred to modify a divorce and includes a pronoun
and verb (they were) that refer unambiguously to their antecedent. men and women.

A incorrect - the singular child does not agree with the plural men and women. Further this
choice incorrectly introduces the when... phrase with occurring [occurring is a progressive
verb indicating continuity in present, however we are concerned with the divorce that
occured when men or women(now) were children], thus illogically making divorce the
grammatical referent of when a child.

B incorrect - replaces child with children but otherwise fails to correct a 's structure and
errors of logic.

C incorrect corrects only the error created by occurring.

E incorrect - includes an incorrect verb tense (has occurred) and replaces wrongly when
with as. Further each was improperly refers to men and women.

66). C is the best answer - An area about the size of Colorado clearly describes a rough
equivalence between the area of Colorado and the area overseen by the companies.

A, B incorrect - the plural verb have does not agree with the singular subject number.

D, E incorrect - unidiomatic; the correct expression is "from x to y".

ShareThisFacebookTweetLinkedInEmailGoogle Pinterest

Posted by Prachi Pareekh at 12:04 AM No comments: Links to this post


Labels: Idiom, modifier error, Subject verb agreement, Tense

Tuesday, September 12, 2006


Gmat Sentence Correction 63, 64

63). Among lower- paid workers, union members are less likely than non union members to be
enrolled in lower- end insurance plans imposing stricter limits on medical services and
requiring doctors to see more patients, and spend less time with each.

A). imposing stricter limits on medical services and requiring doctors to see more patients, and
spend

B). imposing stricter limits on medical services , requiring doctors to see more patients, and
spending

C). that impose stricter limits on medical services, require doctors to see more patients, and
spend
D). that impose stricter limits on medical services and require doctors to see more patients,
spending

E). that impose stricter limits on medical services, requiring doctors to see more patients and
spending

64). one automobile manufacturer has announced plans to increase the average fuel
efficiency of its sport utility vehicles by 25 percent over the next five years, amounting to
roughly five miles per gallon, and representing the first significant change in the fuel
efficiency of any class of passenger vehicle in almost two decades.

A). amounting to roughly five miles per gallon, and representing

B). amounting to roughly five miles per gallon, and it would represent

C). an increase that would amount to roughly five miles per gallon and it would represent

D). an increase that would amount to roughly five miles per gallon and would represent

E). which is an increase amounting to roughly five miles per gallon, representing

Answer -

63). D is the best choice.

A,B incorrect - insurance plans are not imposing anything

C incorrect Plans cant spend time - plans that impose X, (that) require Y and (that) spend
Z

E incorrect - plans that impose X, (that) requiring Y and (that) spending Z. - Lacks
Parallelism and plans can't spend less time

64). D is the best choice.

A,B,E -- incorrect -- What is amounting ? an increase Thus option C or D

C -- incorrect -- it is not required.

ShareThisFacebookTweetLinkedInEmailGoogle Pinterest

Posted by Prachi Pareekh at 1:31 AM No comments: Links to this post


Labels: PARALLELISM
Gmat Sentence Correction 61, 62

61). Although the first pulsar, or rapidly spinning collapsed star, to be sighted was in the
summer of 1967 by graduate student Jocelyn Bell, it had not been announced until February,
1968.

A) Although the first pulsar, or rapidly spinning collapsed star, to be sighted was in the
summer of 1967 by graduate student Jocelyn Bell, it had not been announced until February,
1968.

B) Although not announced until February, 1968, in the summer of 1967 graduate student
Jocelyn Bell observed the first pulsar, or rapidly spinning collapsed star, to be sighted.

C). Although observed by graduate student Jocelyn Bell in the summer of 1967, the discovery
of the first sighted pulsar, or rapidly spinning collapsed star had not been announced before
February, 1968.

D). The first pulsar, or rapidly spinning collapsed star, to be sighted was observed in the
summer of 1967 by graduate student Jocelyn Bell, but the discovery was not announced until
February, 1968.

E) The first sighted pulsar, or rapidly spinning collapsed star, was not announced until
February, 1968, while it was observed in the summer of 1967 by graduate student Jocelyn
Bell.

62). During the same period in which the Maya were developing a heiroglphic system of
writing, the Aztec people also developed a written language, but it was not as highly
sphisticated as that of the Maya and was more pictographic in nature.

A). but it was not as highly sphisticated as that of the Maya and was

B). but it was not as highly sophisticated as the Maya, being

C). but, not as highly sophisticated as the Maya's, was

D). not as highly sphisticated as that of the Maya, however, being

E). not as highly sphisticated as the Maya's however, it was

Answer -

61). Best answer choice is D

A incorrect usage of past perfect tense incorrect here.


B incorrect dangling modifierAlthough not announced. graduate student Jocelyn
Bell

C incorrect dangling modifier observed.. discovery of

E incorrect usage of while is wrong here. Further the sentence conveys the idea that the
pulsar was announced which is wrong sighting was announced instead.

62). A is the best choice.

B incorrect wrong comparison comparing system of writing with Maya. Further options
having being in them must be avoided on GMAT

C, D, E incorrect Subject of the sentence not clear.

E incorrect run-on sentence

ShareThisFacebookTweetLinkedInEmailGoogle Pinterest

Posted by Prachi Pareekh at 1:23 AM 3 comments: Links to this post


Labels: Comparison, modifier error, Tense

Saturday, August 26, 2006


Gmat Sentence Correction 60

60). A star will compress itself into a white dwarf, a neutron star, or a black hole after it passes
through a red giant stage, depending on mass.

(A). A star will compress itself into a white dwarf, a neutron star, or a black hole after it passes
through a red giant stage, depending on mass.

(B). After passing through a red giant stage, depending on its mass, a star will compress itself
into a white dwarf, a neutron star, or a black hole.

(C). After passing through a red giant stage, a star's mass will determine if it compresses itself
into a white dwarf, a neutron star, or a black hole.

(D). Mass determines whether a star, after passing through the red giant stage, will compress
itself into a white dwarf, a neutron star, or a black hole.

(E). The mass of a star, after passing through the red giant stage, will determine whether it
compresses itself into a white dwarf, a neutron star, or a black hole.

Answer -
60). D is the best choice -- Mass determines whether a star, after passing through the red giant
stage, will compress itself into a white dwarf, a neutron star, or a black hole.
(Read the sentence without the words stated in orange color.)

A - incorrect -- dangling modifier...(depending on mass)

B - incorrect -- "its" refers to the red giant stage's mass.

C - incorrect -- star's mass will determine if it compress itself...again here it refers to mass
rather than star.

E - incorrect -- Mass of start after passing....it is star that is passing and not the mass.

ShareThisFacebookTweetLinkedInEmailGoogle Pinterest

Posted by Prachi Pareekh at 12:25 PM No comments: Links to this post


Labels: modifier error, Pronoun error

Gmat Sentence Correction 58, 59

58). There are hopeful signs that we are shifting away from our heavy reliance on fossil fuels:
more than ten times as much energy is generated through wind power now than it was in
1990.

A). generated through wind power now than it was

B). generated through wind power now as it was

C). generated through wind power now as was the case

D). now generated through wind power as it was

E). now generated through wind power than was the case.

59). The computer company has announced that it will purchase the color-printing divison of
a rival company for $ 950 million, which is part of a deal that will make it the largest
manufacturer in the office color-printing market.

A). million, which is part of a deal that will make

B). million, a part of a deal that makes

C). million, part of a deal making

D). million as a part of a deal to make


E). million as part of a deal that will make

Answers -

58). C is correct answer - correct idiom as X as Y

A,B,D incorrect no clear referent for it

E incorrect than is used incorrectly in this option.

59). E is the best choice.

B,C,D incorrect wrong tense usage -- present tense is used instead of future tense

A incorrect which referrent not clear..

Note Always remember a participle phrase as in C here, is used for an event in the past or
which is going on at present but never for the future the reason why again C is wrong. C is
basically changing the meaning of the sentence.

ShareThisFacebookTweetLinkedInEmailGoogle Pinterest

Posted by Prachi Pareekh at 12:17 PM No comments: Links to this post


Labels: Idiom, Pronoun error, Tense

Gmat Sentence Correction 56, 57

56). Some patients who do not respond therapies of depression may simply have received
inadequate treatment, having, for example been prescribed a drug as a dosage too low to be
effective or having been taken off a drug too soon.

(A). having, for example been prescribed a drug as a dosage too low to be effective or having
been

(B). having, for example, a drug prescription that was ineffective because the dosage was too
low, or being

(C). as, for example, having too low of a dosage of prescribed drug for it to be effective, or
being

(D). when they have, for example, been prescribed too low a drug dosage for it to be effective,
or were

(E). for example, when they have a drug prescription with a dosage too low to be effective, or
been

57). Officials at the United States Mint believe that the Sacagawea dollar coin will be used
more as a substitute for four quarters rather than for the dollar bill because its weight, only
8.1 grams, is far less than four quarters, which weigh 5.67 grams each.

(A). more as a substitute for four quarters rather than for the dollar bill because its weight,
only 8.1 grams, is far less than

(B). more as a substitute for four quarters than the dollar bill because it weighs only 8.1
grams, far lighter than

(C). as a substitute for four quarters more than for a dollar bill because it weighs only 8.1
grams, far less than

(D). as a substitute for four quarters more than the dollar bill because its weight of only 8.1
grams is far lighter than it is for
(E). as a substitute more for four quarters rather than for the dollar bill because its weight,
only 8.1 grams, is far less than it is for

Answers -

56). A is the best choice. having.been is correct usage as it maintains parallelism.

B incorrect idea conveyed by this option is that a drug prescription was ineffective which
cannot be true.

C incorrect -- parallelism is violated here having being

D incorrect -- wrong tense and violating parallelism

E incorrect -- violating parallelism

57). C is the best choice.

A,D,E incorrect violating parallelism it should be weighs and not weights

B incorrect incorrectly comparing as a substitute with four quarters

ShareThisFacebookTweetLinkedInEmailGoogle Pinterest

Posted by Prachi Pareekh at 12:06 PM 3 comments: Links to this post


Labels: Comparison, PARALLELISM, Tense
Gmat Sentence Correction 54, 55

54). A new phenomena, which is visible at Managua's major intersections, are waves of
vendors and beggars, which include many children and mob cars at the stoplights.

(A) A new phenomena, which is visible at Managua's major intersections, are waves of
vendors and beggars, which include many children and

(B) Visible at Managua's major intersections are waves of vendors and beggars with many
children, new phenomena that

(C) A new phenomenon visible at Managua's major intersections is waves of vendors and
beggars, many of them children, who

(D) Phenomenally new waves of vendors, beggars, and many children are visible at
Managua's major intersections, which

(E) A wave of vendors and beggars, many of whom are children, are visible at Managua's
major intersections, where they are a new phenomenon and

55). To meet the rapidly rising market demand for fish and seafood, suppliers are growing fish
twice as fast as their natural growth rate, cutting their feed allotment by nearly half and
raising them on special diets.

(A). their natural growth rate, cutting their feed allotment

(B). their natural growth rate, their feed allotment cut

(C). growing them naturally, cutting their feed allotment

(D). they grow naturally, cutting their feed allotment

(E). they grow naturally, with their feed allotment cut

Answers -

54). C is the best choice.

A,E - incorrect -- Subject-Verb Agreemant

B - incorrect -- phenomenon cannot rob cars.

D - incorrect -- very awkward , changes the meaning of the sentence


55). D is the correct answer - parallelism is maintained -"growing....cutting...raising...".
Further antecedent of they is clear i.e the fish

C - incorrect -- after as fast as we need a clause.

A, B - incorrect -- wrong comparison -- cannot compare verb with noun --- growing fish
(verb) with growth rate (noun).

B, E - incorrect -- violating parallelism.

ShareThisFacebookTweetLinkedInEmailGoogle Pinterest

Posted by Prachi Pareekh at 11:48 AM No comments: Links to this post


Labels: Comparison, PARALLELISM, Subject verb agreement

Gmat Sentence Correction 52, 53

52). Byron possessed powers of observation that would have made him a
great anthropologist and that makes his letters as a group the rival of the
best novels of the time.

(A) makes his letters as a group the rival of

(B) makes his letters as a group one to rival

(C) makes his letters a group rivaling

(D) make his letters as a group the rival of

(E) make his letters a group which is the rival of

53). The expected rise in the price of oil could be a serious impact to
industrialized nations and severely diminish the possibility to have an
economy free of inflation.

(A) be a serious impact to industrialized nations and severely diminish the


possibility to have

(B) seriously impact on industrialized nations and severely impede the


possibility to have

(C) seriously impact on industrialized nations and severely impede the


possibility of having

(D) have a serious impact on industrialized nations and severely impede


the possibility to have
(E) have a serious impact on industrialized nations and severely diminish
the possibility of having

Answers -

52). Correct answer - D

A,B,C - incorrect - subject verb agreement - powers....make

E - wordy

53). E is the best choice.

A,B,C - incorrect -- Always remember in GMAT --impact should remain a


noun; a proposal can have an impact.(The only thing that can be impacted
is a wisdom tooth).

D - incorrect - "possibility of" is correct idiom. Further "diminish" instead


of "impede" is more appropriate to use here.

ShareThisFacebookTweetLinkedInEmailGoogle Pinterest

Posted by Prachi Pareekh at 11:43 AM No comments: Links to this post


Labels: Idiom, Subject verb agreement, wordy

Gmat Sentence Correction 50, 51

50). In large doses, analgesics that work in the brain as antagonists to


certain chemicals have caused psychological disturbances in patients,
which may limit their potential to relieve severe pain.

(A) which may limit their potential to relieve

(B) which may limit their potential for relieving

(C) which may limit such analgesics potential to relieve

(D) an effect that may limit their potential to relieve

(E) an effect that may limit the potential of such analgesics for relieving

51). The man was always aware, sometimes proudly and sometimes
resentfully, that he was a small-town Midwesterner who was thrust into a
world that was dominated by wealthier, better-educated, and more
polished people than him.

(A) who was thrust into a world that was dominated by wealthier, better-
educated, and more polished people than him

(B) who had been thrust into a world that was dominated by more
wealthy, educated, and polished people than him

(C) who had been thrust into a world dominated by wealthier, better-
educated, and people more polished than he was

(D) thrust into a world dominated by more wealthy, educated, and


polished people than him

(E) thrust into a world dominated by wealthier, better-educated, and


more polished people than he

Answers -

50). E is the best choice

A,B,C - incorrect - use of "which" ambiguous - referring to 'patients' or


'analgesics' or..

D - incorrect - "their" -ambiguous - referring to 'patients' or 'analgesics' or


effects of the analgesics...

51). E is the correct answer.

A,B,D- incorrect - Subject form of pronoun always comes after than or


as. (Pronoun errors)

C - incorrect - ".. than he was" - was needs to be ellipsed as I mentioned for


Do - auxiliary verb in my GMAT Grammar blog.

ShareThisFacebookTweetLinkedInEmailGoogle Pinterest

Posted by Prachi Pareekh at 11:03 AM No comments: Links to this post


Labels: Pronoun error

Saturday, August 12, 2006


Gmat Sentence Correction 49
49). With a total population of less than two hundred and fewer breeding females than ever
before, the American crocodile seemed a decade ago to be in danger of disappearing.

(A) of less than two hundred and fewer

(B) lower than two hundred and less

(C) lesser than two hundred and fewer

(D) fewer than two hundred and less

(E) of fewer than two hundred and of fewer

Answer -

49). A is the right answer.

D, E - incorrect -- use "fewer" only when referring to countable nouns. Population is very
much like "mass" here and so less/lesser should be used.

B,C - incorrect -- we require 'of ' after population and before less, and "fewer" for refering to
the number of breeding females.

ShareThisFacebookTweetLinkedInEmailGoogle Pinterest

Posted by Prachi Pareekh at 9:53 AM No comments: Links to this post


Labels: less vs fewer

Monday, July 31, 2006


Gmat Sentence Correction 47, 48

47). Most state constitutions now mandate that the state budget be
balanced each year.

(A) mandate that the state budget be balanced

(B) mandate the state budget to be balanced

(C) mandate that the state budget will be balanced

(D) have a mandate for a balanced state budget

(E) have a mandate to balance the state budget

48). Two new studies indicate that many people become obese more due to
the fact that their bodies burn calories too slowly than overeating.

(A) due to the fact that their bodies burn calories too slowly than
overeating

(B) due to their bodies burning calories too slowly than to eating too much

(C) because their bodies bum calories too slowly than that they are
overeaters

(D) because their bodies bum calories too slowly than because they eat too
much

(E) because of their bodies burning calories too slowly than because of
their eating too much

Answers -

47). A is the correct answer - When mandate is used as a verb to mean


"make it mandatory,' it must be followed by that and a verb in the
subjunctive mood, as in A, the best answer: mandate that x be balanced.

B - incorrect -- uses the ungrammatical mandate x to be balanced.

C - incorrect -- inappropriately uses the future indicative, will be, rather


than the subjunctive.

D, E - incorrect -- use wordy and imprecise expressions in place of the verb


mandate: neither have a mandate for a balanced... budget nor have a
mandate to balance the ... budget makes clear that the requirement is
made by the constitution. It is also unclear in D whether each year refers
to the mandating or the balancing.

48). D is the best choice -- The members of a comparison (more X than Y)


should be expressed in parallel form. This choice correctly uses parallel
clauses introduced by because. The clauses themselves are clear and
direct.

E - incorrect -- uses parallel forms, but the convoluted structures are


awkward and wordy. Furthermore, the word "bodies" would need an
apostrophe (bodies') since it is the logical subject of the gerund burning
(that is, it answers the question, "Whose burning?").

A, B, C - incorrect -- do not use parallel forms for the two members of the
comparison. In addition, A and B use "due to" unidiomatically to mean
"because"; properly used, "due to" is synonymous with "attributable to".

ShareThisFacebookTweetLinkedInEmailGoogle Pinterest

Posted by Prachi Pareekh at 11:12 PM No comments: Links to this post


Labels: Comparison, PARALLELISM, subjunctive, wordy

Gmat Sentence Correction 45 , 46

45). The Coast guard is conducting tests to see whether pigeons can be
trained to help find survivors of wrecks at sea.

(A) to see whether pigeons can be trained to help find

(B) to see whether pigeons can be trained as help to find

(C) to see if pigeons can be trained for helping to find

(D) that see if pigeons are able to be trained in helping to find.

(E) that see whether pigeons are able to be trained for help in finding

46). The first decision for most tenants living in a building undergoing being converted to
cooperative ownership
is if to sign a no-buy pledge with the other tenants.

(A) being converted to cooperative ownership is if to sign

(B) being converted to cooperative ownership is whether they should be signing

(C) being converted to cooperative ownership is whether or not they sign

(D) conversion to cooperative ownership is if to sign

(E) conversion to cooperative ownership is whether to sign

Answers -

45). A is the correct answer -- This choice clearly states both the purpose of
the test and the purpose of the training.

B - incorrect -- "As help to find" is unidiomatic.


C - incorrect -- "For helping to find" is not the correct idiom; "whether" is
preferred to "if" when there are only two alternatives.

D - incorrect -- "That see " is ungrammatical; "whether" is preferred to "if"


when there are only two alternatives; "are able to be" should be replaced
by the concise "can"; "in helping to find" should be " to help find".

E - incorrect -- "For helping in finding" is incorrect idiom; "are able to be"


should be replaced by the concise "can".

46). E is the best choice -- idiomatically completes "whether" with an infinitive, "to sign",
that functions as a noun equivalent of decision. It also uses the noun conversion, which
grammatically completes the phrase begun by "undergoing".

A, B, C - incorrect -- the phrase "being converted" is awkward and redundant, since the sense
of process indicated by
"being" has already been conveyed by "undergoing".

A, D - incorrect -- can be faulted for saying "if" rather than "whether", since the sentence
poses alternative possibilities, to sign or not to sign.

ShareThisFacebookTweetLinkedInEmailGoogle Pinterest

Posted by Prachi Pareekh at 10:44 PM No comments: Links to this post


Labels: Idiom, redundancy, Whether vs if

Friday, July 28, 2006


Gmat Sentence Correction 43, 44

43). The Diary of Anne Frank tells the true story of a young girl and her family that were
hidden during the Nazi occupation of the Netherlands by a gentile Dutch couple, though they
were eventually discovered.

A). that were hidden during the Nazi occupation of the Netherlands by a gentile Dutch couple,
though they were eventually discovered

B). that were hidden by a gentile Dutch couple during the Nazi occupation of the Netherlands,
though they were eventually discovered

C). whom a gentile Dutch couple hid during the Nazi occupation of the Netherlands but were
eventually discovered

D). who were hidden by a gentile Dutch couple during the Nazi occupation of the Netherlands
but were eventually discovered

E). who were hidden by a gentile Dutch couple during the Nazi occupation of the Netherlands
even though they were eventually discovered

44). Despite the sequel's poor critical reception, most people seeing it find the acting and
cinematography at least as good or even better than the original.

A). most people seeing it find the acting and cinematography at least as good or even better
than the original

B). most people seeing it find the acting and cinematography at least as good or even better
than the original's

C). most people who see the film find the acting and cinematography at least as good as or
even better than those in the original

D). most people who see it find the acting and cinematography at least as good or even better
than those in the original

E). most people seeing the film find the acting and cinematography at least as good as or even
better than those of the original's

Answers -

43). The correct answer is D -- It correctly uses the pronoun "who" to refer to a girl and her
family. Additionally, the phrase "by a gentile Dutch couple" is placed immediately after "who
were hidden" to clarify the meaning. Finally, the ambiguous pronoun issue is avoided
altogether.

A - incorrect -- The original sentence uses the relative pronoun "that" where "who" is
preferred because the antecedent is a group of people. Also, the prepositional phrase "by a
gentile Dutch couple" is placed in such a way as to suggest that the occupation was carried
out by the couple. Finally, the pronoun "they" is ambiguous it could refer to the family or to
the couple.

B - incorrect -- This choice incorrectly uses the relative pronoun "that" to refer to a group of
people. Additionally, the pronoun "they" is ambiguous - it could refer to the family or to the
couple.

C - incorrect -- The use of the object pronoun "whom" makes "a girl and her family" the
object of the clause "a gentile Dutch couple hid;" however "a girl and her family" are the
subject of the next clause "were eventually discovered." This is a mismatch.

E - incorrect -- The pronoun "they" is ambiguous - it could refer to the family or to the
couple.

44). The correct answer is C -- This choice correctly replaces "it" with "film", adds the
missing "as", and makes clear that the acting and cinematography in the sequel are
compared to the acting and cinematography in the original.

A - incorrect -- First, the pronoun "it" is used to refer to the film, but the film has not been
mentioned in the sentence. Instead "the sequel's poor critical reception" has been mentioned.
This is not the same as the film, so the pronoun "it" has no grammatical antecedent and must
be changed.
Second, the phrase "at least as good or even better than" is incomplete: there should be
another "as" after "good".
Third, the cinematography and acting are being compared to the "original". What is meant
here is that the cinematography and acting in the sequel are as good as the cinematography
and acting in the original. This must be changed.

B - incorrect -- The pronoun "it" has no grammatical antecedent and the phrase "at least as
good or even better" is missing the second "as" after "good."

D - incorrect -- The pronoun "it" has no grammatical antecedent and the phrase "at least as
good or even better" is missing the second "as" after "good."

E - incorrect -- This choice introduces the possessive redundancy "those of the original's."
One could say either "those of the original" or "the original's", but using both is redundant.

ShareThisFacebookTweetLinkedInEmailGoogle Pinterest

Posted by Prachi Pareekh at 1:54 AM 3 comments: Links to this post


Labels: Pronoun error, redundancy

Sunday, July 23, 2006


Gmat Sentence Correction 41, 42

41). Starting at age four, Mozart's father began taking him on tours of the capitals of Europe,
in order to demonstrate his musical talents.

(A). Mozart's father began taking him on tours of the capitals of Europe, in order to
demonstrate his musical talents

(B). Mozart's father began taking the boy on tours of the capitals of Europe, to demonstrate
his musical talents

(C). Mozart began accompanying his father on tours of the capitals of Europe, to demonstrate
his own musical talents
(D). Mozart was accompanying his father on tours of Europe's capitals, to demonstrate his
musical talents

(E). Mozart's father began taking him on tours of the capitals of Europe, to demonstrate the
boy's musical talents

42). According to the professor's philosophy, the antidote to envy is one's own work, always
one's own work: not thinking about it, not assessing it, but simply doing it.

(A) one's own work, always one's own work: not thinking about it, not assessing it, but simply
doing it

(B) always work; because you don't think about it or assess it, you just do it

(C) always one's own work: not thinking about or assessing it, but simply to do it

(D) not to think or assess, but doing one's own work

(E) neither to think about one's own work nor to assess it, it is always simply doing it

Answers -

41). C is the best choice -- "Mozart" is correctly placed as the subject of the opening modifier
"starting at age four." Additionally, the phrase "his own" clarifies that the musical talents in
question are those of the subject, Mozart.

A - incorrect -- The original sentence begins with a modifier ("starting at age four") that
describes Mozart. Yet, the subject of the main clause is Mozart's father. We need to find a
choice that make Mozart himself the subject. Moreover, the pronoun "him" has no
grammatical antecedent, since it is meant to refer to Mozart despite the fact that Mozart is not
actually present in the sentence (we have "Mozart's father" instead). Finally, the pronoun
"his" is somewhat ambiguous: does it refer to Mozart (who, again, is not in the sentence) or
to his father?

B -- incorrect -- "Mozart's father" serves illogically as the subject of the opening modifier
"starting at age four." Additionally, the pronoun "his" is ambiguous in that it could refer to
the boy or the father.

D -- incorrect -- This choice incorrectly uses the verb "was," which does not match the
ongoing nature of the modifier "starting at age four." Additionally, the pronoun "his" is
ambiguous in that it could refer to Mozart or his father.

E -- incorrect -- "Mozart's father" serves illogically as the subject of the opening modifier
"starting at age four." Additionally, the pronoun "him" has no grammatical antecedent, since
it is meant to refer to Mozart despite the fact that Mozart is not actually present in the
sentence (we have "Mozart's father" instead).

42). The correct answer is A - maintains parallelism - thinking, assessing, doing

D, E - incorrect - violates parallelism -- think, assess, doing are not parallel

C - incorrect - violates parallelism -- thinking, assessing and to do it are not parallel.

B - incorrect -- the antidote to envy is to work always -- changes the meaning of the original
sentence.

977). Some archaeologists claim that the tablets found at Ebla could force a revision of
current theories on the origins of Judaism and Christianity, alter many scriptural
interpretations, make all current Bible translations obsolete, and scholars may be required to
credit the Old Testament with greater historical accuracy.

(A) scholars may be required to credit the Old Testament with greater historical accuracy

(B) crediting the Old Testament with greater historical accuracy may be required of scholars

(C) require that scholars are to credit the Old Testament with greater historical accuracy

(D) crediting the Old Testament with greater historical accuracy may be a scholarly
requirement

(E) require scholars to credit the Old Testament with greater historical accuracy

978). Tektites, which may have been propelled to Earth from lunar volcanoes, are much like
the volcanic glass obsidian, but their chemical composition is different than any terrestrial
lava; they contain far less water than obsidian does and none of its characteristic
microcrystals.

(A) is different than any terrestrial lava; they contain

(B) is different than any terrestrial lava's, containing

(C) is different from that of any terrestrial lava; they contain

(D) differs from any terrestrial lava in containing

(E) differs from that of any terrestrial lava's, containing

Answers --
977). Best answer is E

A,B, D - incorrect - violating parallelism

C - incorrect - violating parallelism by inserting unnecessary that

978). C is the best choice.

A, B - incorrect - different than - unidiomatic

E - incorrect - uses repeated possession" that of .....lava's"

either use the "possesive form" or use the construction "that of", but not both.

D - incorrect comparison - compares the lava with the chemical composition of Tektites

ShareThisFacebookTweetLinkedInEmailGoogle Pinterest

Posted by Prachi Pareekh at 12:46 AM No comments: Links to this post


Labels: Comparison, Idiom, PARALLELISM

SC's Question number - 979, 980

979). The failing of the book lies not in a lack of attention to scientific detail but in the
depiction of scenes of life and death in the marine world with emotional overtones that reduce
the credibility of the work.

(A) the depiction of scenes of life and death in the marine world with emotional overtones that

(B) fact that it depicts marine world scenes of life and death as having emotional overtones
that

(C) depiction of scenes of life and death in the marine world, whose emotional overtones

(D) depiction of marine world scenes of life and death, which have emotional overtones and
thus

(E) fact that if depicts scenes of life and death in the marine world, whose emotional overtones

980). The Immigration Service now has the discretionary power to keep families united even
though all their members do not meet the five-year residency requirement.

(A) all their members do not meet the five-year residency requirement

(B) not all their members meet the five-year residency requirement
(C) all their members have not met the requirement for a five-year residency

(D) not all their members have resided for five years, a requirement

(E) all their members have not resided for five years, as required

Answers --

979). A is the best choice

B, E - incorrect - fact is incorrect as this is only an opinion.

D - incorrect - wrongly suggests that life and death have overtones

C - incorrect - unidiomatic - "depict X with Y" - correct idiom

"depict X with Y" - is used to show more of the way in which the depiction is made

980). B is the best choice.

D, E - incorrect - the "five-year residency requirement" as an adjective does not need an


"s" at the end of "five-year".

"s" right in case "five years" was the subject or object of the sentence.

A - incorrect - "X can have discretionary power even though not all members meet some
conditions" - instead of not all should be some

C - incorrect - wrong tense - simple present tense is right

ShareThisFacebookTweetLinkedInEmailGoogle Pinterest

Posted by Prachi Pareekh at 12:38 AM No comments: Links to this post


Labels: Idiom, Tense

SC's Question number - 981, 982

981). The Lake Manyara Park in Tanzania affords the visitor with unequaled opportunities to
photograph lions playing in trees without the aid of telephoto lenses.

(A) The Lake Manyara Park in Tanzania affords the visitor with unequaled opportunities to
photograph lions playing in trees without the aid of telephoto lenses.

(B) The Lake Manyara Park in Tanzania permits the visitor unequaled opportunities to
photograph lions playing in trees without the aid of telephoto lenses.

(C) The Lake Manyara Park in Tanzania gives the visitor the unequalled opportunity to
photograph lions playing in trees without telephoto lenses.

(D) The visitor to the Lake Manyara Park in Tanzania has the unequaled opportunity to
photograph lions playing in trees without the aid of telephoto lenses.

(E) Even without the aid of telephoto lenses, the visitor to Tanzania's Lake Manyara Park has
an unequaled opportunity to photograph lions playing in trees.

982). The rise in the price of crabmeat and an increase in demand has convinced some
Floridians they should try to harvest and sell a species of large crab that lives deep in the
waters of the Gulf of Mexico.

(A) has convinced some Floridians they should try to harvest and sell a species of large crab

(B) has convinced some Floridians to try harvesting and selling a large crab species

(C) has convinced some Floridians that they should try the harvest and sale of a large crab
species

(D) have convinced some Floridians to try to harvest and sell a species of large crab

(E) have convinced some Floridians to try and harvest and sell a large crab species

Answers --

981). Best choice is E

A, B, C, D - incorrect - modifier error - wrongly suggests -- lions playing in trees without the
aid of telephoto lenses.

982). Best choice is D

A, B, C - incorrect - should be have not has -The rise in the price of crabmeat and an
increase demand ...have

E - incorrect - awkward construction -- "have convinced some Floridians to try and harvest
and sell a large crab species"

ShareThisFacebookTweetLinkedInEmailGoogle Pinterest

Posted by Prachi Pareekh at 12:29 AM No comments: Links to this post


Labels: modifier error, Subject verb agreement
Thursday, February 01, 2007
SC's Question number - 983, 984

983). The skill and the precision of the Anasazi, ancient inhabitants of the Southwest, in
measuring the movements of the Sun and Moon is evidenced not only at Chaco Canyon but at
a number of other sites.

(A) in measuring the movements of the Sun and Moon is evidenced not only at

(B) in measuring the movements of the Sun and Moon are evidenced not only at

(C) in measuring the movements of the Sun and Moon is evidenced at not only

(D) to measure the movements of the Sun and Moon is evidenced at not only

(E) to measure the movements of the Sun and Moon are evidenced not only at

984). There is ample evidence, derived from the lore of traditional folk medicine, that
naturally occurring antibiotics are usually able to be modified to make them a more effective
drug.

(A) are usually able to be modified to make them a more effective drug

(B) are usually able to be modified to make them more effective drugs

(C) are usually able to be modified, which makes them more effective drug

(D) can usually be modified to make them a more effective drug

(E) can usually be modified to make them more effective drugs

Answers --

983). B is the best choice.

A, C, D - incorrect - Plural subject so we need are not is

E - incorrect - Idiom: skill in doing something.


His skill in teaching is better than His skill to teach

984). E is the best choice.

A, B, C - incorrect - "able to be" - On GMAT if active voice is correct, it is preferred over


passive voice
D - incorrect - violates parallelism - must be ......antibiotics .... drugs

ShareThisFacebookTweetLinkedInEmailGoogle Pinterest

Posted by Prachi Pareekh at 11:47 PM No comments: Links to this post


Labels: Idiom, PARALLELISM, Passive, Subject verb agreement

SC's Question number - 985, 986

985). Those who have visited the Grand Canyon have typically seen layers of sediment in the
gaping canyon, with different colors that mark the passage of time like the rings in a tree
trunk.

(A) seen layers of sediment in the gaping canyon, with different colors that mark

(B) see layers of sediment in the gaping canyon, whose different colors mark

(C) been seeing layers of sediment in the gaping canyon, whose different colors are markers
of

(D) been able to see layers of sediment in the gaping canyon, with different colors marking

(E) seen layers of sediment in the gaping canyon, marking by different colors

986). Turkey's economy has grown prodigiously over the past ten years, averaging an annual
increase each year that is about eight percentone of the highest growth rates in the world.

(A) averaging an annual increase each year that is about eight percent

(B) with an annual average each year which is about eight percent

(C) eight percent is what it averages out to each year

(D) with an average annual increase of about eight percent

(E) the average of the annual increase each year is about eight percent

Answers --

985). OA - A, the answer given in 1000SC's collection is wrong.

B, C - incorrect - no clear antecedent for pronoun whose

D - incorrect - wordy - been able to see


E - incorrect - unclear subject for marking - layers or the Canyon that is marking the
passage of time?

986). OA - D

A, B, E - incorrect - redundancy error - average annual increase and each year

C - incorrect - awkward construction

ShareThisFacebookTweetLinkedInEmailGoogle Pinterest

Posted by Prachi Pareekh at 11:42 PM No comments: Links to this post


Labels: Pronoun error, redundancy, wordy

SC's Question number - 987, 988

987). With cloning technology, scientists are approaching what has long been the ultimate
goal of modern husbandry: achieving a consistency of quality and production in farm animals
as once thought to be limited to manufactured goods.

(A) achieving a consistency of quality and production in farm animals as once thought to be

(B) achieving farm animals with a consistency of quality and production as were once thought
of as

(C) achieving in farm animals a consistency of quality and production that was once thought
to be

(D) achievement of farm animals whose consistency of quality and production are the same as
what were once thought to he

(E) achievement of farm animals at a consistency of quality and production once thought of
as

988). Tom Bradley was mayor of Los Angeles from 1973 to 1993, an era when the city had
transformed from a collection of suburban neighborhoods to the second-largest city in the
United States.

(A) an era when the city had transformed

(B) an era during which the city was transformed

(C) an era that transformed it


(D) during which era the city transformed

(E) during which the city was transformed

Answers --

987). C is the best choice.

A - incorrect - is unclear -- "consistency of quality" and "production in farm animals" or


consistency of "quality and production", in farm animals.

A, B - incorrect - Actions are not being compared so use of AS is wrong in A

D, E - incorrect - "achievement of farm animals" - making no sense.

B, D - incorrect - using plural verb for Consistency - were

988) . B is the best choice - (Appositive) -- Emphasis is on the ERA, when Y was Mayor.

A - incorrect - Cannot be past and past-perfect at the same time - "the city has
transformed"

C - incorrect - use of it - ambiguous

D, E - incorrect - use of "during" is wrong in D and E because there is no antecedent for


"during which" .

We need 'an era' as an appositive otherwise the clauses at the end would modify the year
instead of the period

ShareThisFacebookTweetLinkedInEmailGoogle Pinterest

Posted by Prachi Pareekh at 11:35 PM No comments: Links to this post


Labels: Comparison, Pronoun error, Tense

SC's Question number - 989, 990

989). The results of the company's cost-cutting measures are evident in its profits, which
increased five percent during the first three months of this year after it fell over the last two
years.

(A) which increased five percent during the first three months of this year after it fell

(B) which had increased five percent during the first three months of this year after it had
fallen
(C) which have increased five percent during the first three months of this year after falling

(D) with a five percent increase during the first three months of this year after falling

(E) with a five percent increase during the first three months of this year after having fallen

990). Part of the proposed increase in state education spending is due to higher enrollment,
since the number of students in public schools have grown steadily since the mid-1980's and,
at nearly 47 million, are at a record high.

(A) enrollment, since the number of students in public schools have grown steadily since the
mid-1980's and, at nearly 47 million, are at

(B) enrollment, with a number of students in public schools growing steadily since the mid-
1980's and, at nearly 47 million, reaching

(C) enrollment: since students in public schools have grown steadily in number since the mid-
1980's and, at nearly 47 million, have reached

(D) enrollment: the number of student in public schools has grown steadily since the mid-
1980's and, at nearly 47 million, has reached

(E) enrollment: students in public schools have grown steadily in number, since the mid-
1980's and, at nearly 47 million, are at

Answers:

989). OA - C

A, B - incorrect - wrong tense. Present perfect tense is required here.

D, E - incorrect - non essential modifier which is needed to modify profits

990). OA - D

A - incorrect - subject verb agreement - ...the number(singular)...have(plural)..

B - incorrect - awkward, violating paarllelism

A, C - incorrect - use of since is wrong.

C, E - incorrect - suggests that students have reached a record high..it is the number of
students that has reached a record high

ShareThisFacebookTweetLinkedInEmailGoogle Pinterest
Posted by Prachi Pareekh at 11:32 PM No comments: Links to this post
Labels: Subject verb agreement, Tense

SC's Question number - 991, 992

991). Vivien Thomas, who had no formal medical training, in struggling


against overwhelming odds, he became a cardiac surgeon and eventually
to receive an honorary doctorate from Johns Hopkins University.

A) who had no formal medical training, in struggling against


overwhelming odds, he became

B) having had no formal medical training, in struggling against


overwhelming odds to become

C) who, having no formal medical training, he struggled against


overwhelming odds in becoming

D) who, having had no formal medical training and struggled against


overwhelming odds, became

E) who had no formal medical training, struggled against overwhelming


odds to become

992). The yield of natural gas from Norway's Troil gas field is expected to
increase annually until the year 2005 and then to stabilize at six billion
cubic feet a day, which will allow such an extraction rate at least for 50
years' production.

A) 2005 and then to stabilize at six billion cubic feet a day, which will
allow such an extraction rate at least for

B) 2005 and then to stabilize at six billion cubic feet a day, an extraction
rate that will allow at least

C) 2005 and then stabilizing at six billion cubic feet a day, with such an
extraction rate at the least allowing

D) 2005, then stabilizing at six billion cubic feet a day, allowing such an
extraction rate for at least

E) 2005, then stabilizing at six billion cubic feet a day, which will allow
such an extraction rate for at least

Answers --

991). E is the best choice - maintains parallelism.


992). B is the best choice

A - incorrect - use of which - ambiguous

C,D,E - incorrect - violating parallelism - to increase .... to stabilize

ShareThisFacebookTweetLinkedInEmailGoogle Pinterest

Posted by Prachi Pareekh at 11:16 PM No comments: Links to this post


Labels: PARALLELISM, Pronoun error

Sunday, January 21, 2007


SC's Question number - 993, 994

993). In archeology, there must be a balance between explanation of the value and workings
of archeology, revealing the mysteries of past and present cultures, and to promote respect for
archaeological sites.

A) between explanation of the value and workings of archeology, revealing the mysteries of
past and present cultures, and to promote

B) among explaining the value and workings of archeology, revealing the mysteries of past
and present cultures, and promoting

C) between explaining the value and workings of archeology, revealing the mysteries of past
and present cultures, and when promoting

D) among explaining the value and workings of archeology, the revelation of the mysteries of
past and present cultures, and to promote

E) between explaining archeology' s value and workings, in the revealing the mysteries of past
and present cultures, and in promoting

994). According to a 1996 survey by the National Association of College an University


Business Officers, more than three times as many independent institutions of higher
education charge tuition and fees of under $8,000 a year than those that charge over $16,000.

A) than those that charge

B) than are charging

C) than to charge
D) as charge

E) as those charging

Answers --

993). B is the best choice - maintains parallelism - among explaining the value and workings
of archaeology, revealing the mysteries of past and present cultures, and promoting
A, C, E - incorrect - incorrectly uses "between" - "Among" is used to compare three or
more things...
D - incorrect - violates parallelism - among explaining the value and workings of
archaeology, the revelation of the mysteries of past and present cultures, and to promote

994). E is the best choice - correct idiom- ..more than three times as many X charge Y as
those charging Z..

ShareThisFacebookTweetLinkedInEmailGoogle Pinterest

Posted by Prachi Pareekh at 10:42 PM No comments: Links to this post


Labels: Comparison, Idiom, PARALLELISM

SC's Question number - 995, 996

995). Nobody knows exactly how many languages there are in the world, partly because of the
difficulty (of? ) distinguishing between a language and the sub-languages or dialects within it,
but those who have tried to count typically have found about five thousand.

A) and the sub-languages or dialects within it, but those who have tried to count typically have
found

B) and the sub-languages or dialects within them, with those who have tried counting typically
finding

C) and the sub-languages or dialects within it, but those who have tried counting it typically
find

D) or the sub-languages or dialects within them, but those who tried to count them typically
found

E) or the sub-languages or dialects within them, with those who have tried to count typically
finding

996). Unlike most severance packages, which require workers to stay until the last day
scheduled to collect, workers at the automobile company are eligible for its severance package
even if they find a new job before they are terminated.
A) the last day scheduled to collect, workers at the automobile company are eligible for its
severance package

B) the last day they are scheduled to collect, workers are eligible for it at the automobile
company's severance package

C) their last scheduled day to collect, the automobile company offers its severance package to
workers.

D) their last scheduled day in order to collect, the automobile company's severance package is
available to workers.

E) the last day that they are scheduled to collect, the automobile company's severance package
is available to workers.

Answers --

995). A is the best choice.

D, E - incorrect - "between A and B" - is the correct idiom.

B - incorrect - Plural "them" is incorrect. (a language - singular)

C - incorrect - "try ~ ing" is different from "try + to +Verb" . Here "try+to" should be used.

Note - the object of "count" is same as that of "knows", therefore it can be ommitted in
Choice A.

996). D is the best choice according to me though 1000 SC's state E to be the best choice.

A, B, C - incorrect - incorrect comparison - workers are compared with package - comparison


must be between "most severance packages" and "the automobile company's severance
package."

E - incorrect - changes the meaning - "last day that they are scheduled to collect" creates
confusion."To collect" is misplaced. E says "the last day that they are scheduled to collect,"
suggesting illogically that workers can be scheduled to collect a day.

Note - D uses "in order to." It is true that "in order to" is often considered wordy -- usually
just "to" is enough. But "in order to" is correct here and it alone is not enough to eliminate
an answer choice.
ShareThisFacebookTweetLinkedInEmailGoogle Pinterest

Posted by Prachi Pareekh at 10:33 PM No comments: Links to this post


Labels: Comparison, Idiom, modifier error, wordy

SC's Question number - 997, 998

997). Studying the fruit fly, a household nuisance but a time-honored experimental subject,
has enabled the secrets of how embryos develop to begin to be unraveled by scientists.

A) Studying the fruit fly, a household nuisance but a time-honored experimental subject, has
enabled the secrets of how embryos develop to begin to be unraveled by scientists.

B) By the study of the fruit fly, a household nuisance and also a time-honored experimental
subject, it was possible for the secrets of how embryos develop to begin to be unraveled by
scientists.

C) By studying a household nuisance but a time-honored experimental subject, the fruit fly
enabled scientist to begin to unravel the secrets of how embryos develop.

D) By studying the fruit fly, a household nuisance and also a time-honored experimental
subject, the secrets of how embryos develop are beginning to be unraveled by scientist.

E) The study of the fruit fly, a household nuisance but a time-honored experimental subject,
has enabled scientist to begin to unravel the secrets of how embryos develop.

998). The Industrial Revolution, making it possible to mass-produce manufactured goods, was
marked by their use of new machines, new energy sources, and new basic materials.

A) making it possible to mass-produce manufactured goods, was marked by their use of

B) making possible the mass production of manufactured goods, marked by the use of

C) which made it possible that manufactured goods were mass-produced, was marked by their
using

D) which made possible the mass-production of manufactured goods, was marked by the use
of

E) which made the mass production of manufactured goods possible and was marked by using

Answers --

997). E is the best choice.


A - incorrect - a household nuisance cannot study about the house fly.

B,C,D - incorrect - are passive.

998). D is the best choice.

A, C - incorrect - antecedent of their not clear.

B - incorrect - distorts the meaning.

ShareThisFacebookTweetLinkedInEmailGoogle Pinterest

Posted by Prachi Pareekh at 10:26 PM 2 comments: Links to this post


Labels: Passive, Pronoun error

SC's Question number - 1000, 999

1000). Japan's abundant rainfall and the typically mild temperature


throughout most of the country have produced a lush vegetation cover
and, despite the mountainous terrain and generally poor soils, it has
made possible the raising of a variety of crops.

A) it has made possible the raising of

B) has made possible fro them to raise

C) have made it possible to raise

D) have made it possible for raising

E) thus making it possible for them to raise

999). In January 1994 an oil barge ran aground off the coast of San Juan,
Puerto Rico, leaking its cargo of 750000 gallons into the ocean, while
causing the pollution of the city's beaches.

A) leaking its cargo of 750000 gallons into the ocean, while causing the
pollution of

B) with its cargo of its cargo of 750000 gallons leaking into the ocean, and
it polluted

C) and its cargo of its cargo of 750000 gallons leaked into the ocean,
polluting
D) while it leaked its cargo of 750000 gallons into the ocean and caused
the pollution of

E) so that its cargo of 750000 gallons leaked into the ocean, and they were
polluting

Answers -

1000). C is the best choice.

A, B - incorrect - Subject "Japan's abundant rainfall and the typically mild temperature" is
plural and hence `have' is required as plural verb.

E - incorrect - violating parallelism -- making . Verb forms for a particular subject in a


sentence should be similar --- produced.... made

D - incorrect - wrong idiom - raising of is different meaning from to raise and is not a correct
idiom here.

999). C - this question I personally feel has a typing mistake.....it should be and its cargo of
750000 gallons leaked into the ocean, polluting ... hence ignoring the typo, least bad answer is
C ---

A - incorrect - use of while incorrect

ShareThisFacebookTweetLinkedInEmailGoogle Pinterest

Posted by Prachi Pareekh at 10:10 PM 2 comments: Links to this post


Labels: Idiom, PARALLELISM, Subject verb agreement

Wednesday, December 13, 2006


Gmat Sentence Correction 95, 96

95). For members of the seventeenth-century Ashanti nation in Africa, animal-hide shields
with wooden frames were essential items of military equipment, a method to protect
warriors against enemy arrows and spears.

(A) a method to protect

(B) as a method protecting

(C) protecting

(D) as a protection of
(E) to protect

96). Scientists have observed large concentrations of heavy-metal deposits in the upper twenty
centimeters of Baltic Sea sediments, which are consistent with the growth of industrial activity
there.

(A) Baltic Sea sediments, which are consistent with the growth of industrial activity there

(B) Baltic Sea sediments, where the growth of industrial activity is consistent with these
findings

(C) Baltic Sea sediments, findings consistent with its growth of industrial activity

(D) sediments from the Baltic Sea, findings consistent with the growth of industrial activity in
the area

(E) sediments from the Baltic Sea, consistent with the growth of industrial activity there

Answers --

95). C is the best choice.


A, B -- incorrect - subject verb agreement - singular "method" refers to the plural "items of
military equipment."
D -- incorrect - "protection" cannot substitute for any prior noun in the sentence. Also this
choice ends the sentence with a prepositional phrase which is incorrect.
E -- incorrect - run on sentence - has a comma splice splitting at the end two independent
clauses making it a run-on sentence.

96). D is the best choice.


A -- incorrect - relative pronoun "which" incorrectly refers to "Baltic Sea sediments"
B -- incorrect - "these" refers to what?
C -- incorrect - "its" has no clear referrent
E -- incorrect - What is consistent with the growth of
industrial activity there ? The Baltic sea sediments ? The large concentrations of heavy metal
deposits?

ShareThisFacebookTweetLinkedInEmailGoogle Pinterest

Posted by Prachi Pareekh at 10:30 AM No comments: Links to this post


Labels: Pronoun error, Subject verb agreement

Gmat SEntence Correction 93, 94

93). As a baby emerges from the darkness of the womb with a rudimentary sense of vision, it
would be rated about 20/500. or legally blind if it were an adult with such vision.
(A) As a baby emerges from the darkness of the womb with a rudimentary sense of vision, it
would be rated about 20/500, or legally blind if it were an adult with such vision.

(B) A baby emerges from the darkness of the womb with a rudimentary sense of vision that
would be rated about 20/500, or legally blind as an adult

(C) As a baby emerges from the darkness of the womb, its rudimentary sense of vision would
be rated about 20/500; qualifying it to be legally blind if an adult

(D) A baby emerges from the darkness of the womb with a rudimentary sense of vision that
would be rated about 20/500; an adult with such vision would be deemed legally blind.

(E) As a baby emerges from the darkness of the womb, its rudimentary sense of vision, which
would deemed legally blind for an adult, would be rated about 20/500.

94). The cameras of the Voyager II spacecraft detected six small, previously unseen moons
circling Uranus, which doubles to twelve the number of satellites now known as orbiting the
distant planet

(A) which doubles to twelve the number of satellites now known as orbiting

(B) doubling to twelve the number of satellites now known to orbit

(C) which doubles to twelve the number of satellites now known in orbit around

(D) doubling to twelve the number of satellites now known as orbiting

(E) which doubles to twelve the number of satellites now known that orbit

Answers --

93). D is the best choice.


A -- incorrect - seems to state that the newborn baby, rather than its sense of vision, would be
rated 20/500.
B, E -- incorrect - use awkward and ambiguous phrasing that suggests that the sense of vision,
rather than an adult with 20/500 vision, would be considered legally blind.
C -- incorrect - incorrectly uses the semicolon, which should separate independent clauses, to
set off a verb phrase. The phrase if an adult in C is also illogical, since it states that a baby
could also be an adult.

94). B is the best choice.


A, C, E -- incorrect - "which" wrongly refers to "Uranus"
D -- incorrect - "known to": when we want to talk about some fact that people have learned
"known as": when we want to talk about the name of somebody or something or its
definition ... hence D is wrong.
ShareThisFacebookTweetLinkedInEmailGoogle Pinterest

Posted by Prachi Pareekh at 10:09 AM No comments: Links to this post


Labels: Idiom, Pronoun error

Gmat Sentence Correction 91, 92

91). Since 1981, when the farm depression began, the number of acres overseen by
professional farm-management companies have grown from 48 million to nearly 59 million,
an area that is about Colorado's size.

(A) have grown from 48 million to nearly 59 million, an area that is about Colorado's size

(B) have grown from 48 million to nearly 59 million, about the size of Colorado

(C) has grown from 48 million to nearly 59 million, an area about the size of Colorado

(D) has grown from 48 million up to nearly 59 million, an area about the size of Colorado's

(E) has grown from 48 million up to nearly 59 million, about Colorado's size

92). Some bat caves, like honeybee hives, have residents that take on different duties such as
defending the entrance, acting as sentinels and to sound a warning at the approach of danger,
and scouting outside the cave for new food and roosting sites.

(A) acting as sentinels and to sound

(B) acting as sentinels and sounding

(C) to act as sentinels and sound

(D) to act as sentinels and to sound

(E) to act as a sentinel sounding

Answers --

91). C is the best choice.


A, B -- incorrect - subject verb agreement - plural verb "have" does not agree with singular
subject "number".
D, E -- incorrect - unidiomatic - "up to" is wrong, correct idiom is "from x to y"

92). B is the best choice -- maintaining parallelism required among and within the three main
verb phrases - defending, acting... sounding, and scouting
A, C, D, E -- incorrect - violating parallelism.
ShareThisFacebookTweetLinkedInEmailGoogle Pinterest

Posted by Prachi Pareekh at 9:43 AM No comments: Links to this post


Labels: Idiom, PARALLELISM, Subject verb agreement

Wednesday, November 08, 2006


Gmat Sentence Correction 89, 90

89). John Smith provides information on the conditions that lead women to a gynecologist,
and he notes that these conditions can, and sometimes are, used in the promotion of surgery
which is not needed.

(A) and he notes that these conditions can, and sometimes are, used in the promotion of
surgery which is not needed

(B) noting that these conditions can be, and sometimes are, used to promote needless surgery

(C) and notes that they could and sometimes are used unnecessarily in promoting surgery

(D) sometimes promoting needless surgery

(E) which they use sometimes to promote unnecessary surgery

90). Machines replacing human labor, there was wide anticipation that the workweek would
continue to become shorter.

(A) Machines replacing human labor, there was wide anticipation that

(B) When machines replaced human labor, there was wide anticipation

(C) As machines replaced human labor, it was widely anticipated that

(D) Insofar as machines replaced human labor, it was widely anticipated

(E) Human labor being replaced by machines, there was wide anticipation that

Answers --

89). B is the best choice.


A - incorrect - wordy...Further "can be (used) and sometimes are used"..omits "be"
C, E - incorrect - ambiguous "they"
D - incorrect - who promotes needless surgery ?...not clear....Conditions or John Smith?

90). Best choice C - only choice to use past tense correctly throughout.
Why It is right in this choice, the link below explains ...
http://gmat-grammar.blogspot.com/2006/07/it-as-subject-of-sentence.html

ShareThisFacebookTweetLinkedInEmailGoogle Pinterest

Posted by Prachi Pareekh at 12:21 AM No comments: Links to this post


Labels: Pronoun error, Tense, wordy

Tuesday, November 07, 2006


Gmat Sentence Correction 87, 88

87). It takes a deft balance between savings discipline, investment knowledge, risk taking,
luck, and time to raise a million dollars through investments.

(A) It takes a deft balance between

(B) Deft balancing is needed between

(C) Deftly balanced, it takes

(D) It takes a deft balance of

(E) A deft balance is what one needs among

88). Last year, land values in most parts of the pinelands rose almost so fast, and in some parts
even faster than what they did outside the pinelands.

(A) so fast, and in some parts even faster than what they did

(B) so fast, and in some parts even faster than, those

(C) as fast, and in some parts even faster than, those

(D) as fast as, and in some parts even faster than, those

(E) as fast as, and in some parts even faster than what they did

Answers --

87). D is the best choice.


A, B - incorrect - usage of between is wrong here...."between" is used when we consider only
two objects. Further correct idiom is "balance of"
C - incorrect - Awkward.....Deftly balanced seems to modify the full sentence
E - incorrect - Changes the meaning (deft balance among a, b and c ) ---"among" incorrect
here, we need "of" here.
"Deft balance of" is not same as "Deft balance among"
E suggests that "Deft balance" is the only required object among all the other objects
"savings discipline, investment knowledge, risk taking, luck and time" ...which is incorrect.

88). D - correct - idiomatic


Comparison in this sentence is between the most parts of pineland and parts outside the
pineland.
A, B, C - incorrect - correct idioms are "as...as" and "so..that"
E - incorrect - "they" referring to what ?

ShareThisFacebookTweetLinkedInEmailGoogle Pinterest

Posted by Prachi Pareekh at 11:46 PM No comments: Links to this post


Labels: Comparison, Idiom, Pronoun error

Monday, November 06, 2006


Gmat Sentence Correction 85, 86

85). Defense attorneys have occasionally argued that their clients' misconduct stemmed from
a reaction to something ingested, but in attributing criminal or delinquent behavior to some
food allergy, the perpetrators are in effect told that they are not responsible for their actions.

(A) in attributing criminal or delinquent behavior to some food allergy

(B) if criminal or delinquent behavior is attributed to an allergy to some food

(C) in attributing behavior that is criminal or delinquent to an allergy to some food

(D) if some food allergy is attributed as the cause of criminal or delinquent behavior

(E) in attributing a food allergy as the cause of criminal or delinquent behavior

86). The voluminous personal papers of Thomas Alva Edison reveal that his inventions
typically sprang to life not in a flash of inspiration but evolved slowly from previous works.

(A) sprang to life not in a flash of inspiration but evolved slowly

(B) sprang to life not in a flash of inspiration but were slowly evolved

(C) did not spring to life in a flash of inspiration but evolved slowly

(D) did not spring to life in a flash of inspiration but had slowly evolved

(E) did not spring to life in a flash of inspiration but they were slowly evolved
Answers --

85). B is the correct answer.


A, C, E - incorrect - "in attributing criminal or delinquent behavior" is modifying
perpetrators which is not correct because the defense attorneys are attributing behavior to
food allergies.
D - incorrect - doesn't identifies the attributors at all.

86). C is the correct answer - Parallelism - ...not...but..


A, B - incorrect - violating parallelism - right form...not in....but in...
D - incorrect - wrong tense
E - incorrect - passive

ShareThisFacebookTweetLinkedInEmailGoogle Pinterest

Posted by Prachi Pareekh at 12:09 AM 1 comment: Links to this post


Labels: modifier error, Tense

Sunday, November 05, 2006


Gmat Sentence Correction 83, 84

83). Aho, a Kiowa matriarch, held festivals in her home, they featured the preparation of great
quantities of ceremonial food, the wearing of many layers of colorful clothing adorned with
silver, and the recounting of traditional tribal jokes and stories.

(A) Aho, a Kiowa matriarch, held festivals in her home, they featured

(B) Festivals were held in Aho, a Kiowa matriarchs home, which featured

(C) Aho, who was a Kiowa matriarch in her home, held festivals featuring

(D) In her home, Aho, a Kiowa matriarch, held festivals that featured

(E) Aho, a Kiowa matriarch, held festivals in her home that featured

84). An array of tax incentives has led to a boom in the construction of new office buildings;
so abundant has capital been for commercial real estate that investors regularly scour the
country for areas in which to build.

(A) so abundant has capital been for commercial real estate that

(B) capital has been so abundant for commercial real estate, so that

(C) the abundance of capital for commercial real estate has been such,
(D) such has the abundance of capital been for commercial real estate that

(E) such has been an abundance of capital for commercial real estate,

Answers --

83). D is the best answer.


A - incorrect - referent of "they" is not present or in other words is unclear
B - incorrect - Aho, a Kiowa matriarchs home, makes it wrong as Aho is Kiowa matriarch and
not the home
C - incorrect - is unnecessarily wordy -- "who was" part
E - incorrect - ambiguous - not clear whether the festival or the home featured the
preparation.

84). Choice A is the correct answer.


B - incorrect - there is repetition of "so" as such it is illogical,
C, E - incorrect - "that" has been omitted which makes both the choices grammatically
incorrect.
D - incorrect - seperation of "has" and "been" is vague and incorrect.
C, D ,E - incorrect - "such" seems to convey the meaning "of a kind" and it is not stressing
"abundant" as it should do otherwise.

ShareThisFacebookTweetLinkedInEmailGoogle Pinterest

Posted by Prachi Pareekh at 11:54 PM No comments: Links to this post


Labels: Pronoun error, wordy

Gmat Sentence Correction 81, 82

81). Despite the recent election of a woman to the office of prime minister, the status of women
in Pakistan is little changed from how it was in the last century.

(A) is little changed from how it was

(B) is a little change from how it was

(C) has changed little

(D) has changed little from how it has been

(E) is little changed from the way it was

82). Although the lesser cornstalk borer is widely distributed, control of them is necessary only
in the South.
(A) the lesser cornstalk borer is widely distributed, control of them is

(B) widely distributed, measures to control the lesser cornstalk borer are

(C) widely distributed, lesser cornstalk borer control is

(D) the lesser cornstalk borer is widely distributed, measures to control it are

(E) it is widely distributed, control of the lesser cornstalk borer is

Answers --

81). C is the best choice.


A, B, D, E - incorrect - "it" referring to what? is it the status of women in the last century or
something else in the last century that we compare the current women status with.
A, B, E - incorrect - "little changed" is incorrect..(little changed means changed to some
extent but not
much....whereas...changed little implies did not changed at all)
B - incorrect - use of "is" (present tense) is incorrect
D - incorrect - sentence says "last century" so "how it has been" is unnecessary to state.

82). D is the best choice.


A - incorrect - "them" is refering to "lesser cornstalk borer" so must be singular.
B - incorrect - "widely distrbuted" is modifying "measures" , should modify "the lesser
cornstalk borer".
C - incorrect - changing the meaning - suggests that lesser cornstalk borer control is widely
distributed instead of lesser cornstalk borer.
E - incorrect - "it" referring to what ? "lesser cornstalk borer"or "control"

939). With its abundance of noun inflections, Icelandic is one of several


Germanic languages that is compact when written but can lengthen
considerably when translated into English.

(A) is compact when written but can lengthen considerably when


translated into English

(B) are compact when they are written, but they can lengthen
considerably when they are translated in English

(C) is compact when written but can lengthen considerably when being
translated into English

(D) are compact when written but can lengthen considerably in English
translation

(E) is compact when it is written but can lengthen considerably when


translated in English

940). With its plan to develop seven and a half acres of shore land,
Cleveland is but one of a large number of communities on the Great Lakes
that is looking to its waterfront as a way to improve the quality of urban
life and attract new businesses.

(A) is looking to its waterfront as a way to improve the quality of urban


life and attract

(B) is looking at its waterfront to improve the quality of urban life and attract

(C) are looking to their waterfronts to improve the quality of urban life
and attract

(D) are looking to its waterfront as a way of improving the quality of


urban life and attracting

(E) are looking at their waterfronts as a way they can improve the quality of urban life and
attract

Answers --

939). OA - D - one of + Plural noun + that/who + Plural verb

A, C, E - incorrect - is wrong - several Germanic languages

B - incorrect - wordy. Further there is no need of first they - when they are written, but they

940). Best answer - C - one of + Plural noun + that/who + Plural verb

A, B - incorrect - is wrong - a large number of communities on the Great lakes - plural (a


large number - always plural)

D - incorrect - subject verb agreement - are ....its

E - incorrect - wrong idiom use look at - look to (someone or something) implies To rely on,
turn to or refer to them or it Example: looked to her for support. look at implies To turn the
eyes in a certain direction so as to see; to use one's sight.Example: Take a look at that new
building

ShareThisFacebookTweetLinkedInEmailGoogle Pinterest

Posted by Prachi Pareekh at 9:44 PM No comments: Links to this post


Labels: Idiom, Subject verb agreement, wordy
SC's Question number - 941, 942

941). With just several quick strokes of the pen, the monkeys were drawn by the artist,
capturing their antics.

(A) the monkeys were drawn by the artist, capturing their antics

(B) the artist sketched the monkeys, capturing their antics

(C) the artist captured the antics of the monkeys, sketching them

(D) the artist sketched the monkeys and also capturing their antics

(E) the monkeys and heir antics were sketched by the artist

942). With only 5 percent of the world's population, United States citizens consume 28 percent
of its nonrenewable resources, drive more than one-third of its automobiles, and use 21 times
more water per capita than Europeans do.

(A) With

(B) As

(C) Being

(D) Despite having

(E) Although accounting for

Answers --

941). Best answer B.

A, E - incorrect - passive. Further the modifier 'With just several quick strokes of the pen'
should be followed by the noun that performs this action.

C - incorrect - ambiguous them, misplaced modifier - monkeys cannot sketch

D - incorrect - wrongly suggests that the artist captured their antics

942). OA - E -- Subject - "United States citizens " and not "United States".

A, C, B - incorrect - need a contrast.

D - incorrect - unidiomatic - despite having.


Further A, D incorrectly modifying United States

ShareThisFacebookTweetLinkedInEmailGoogle Pinterest

Posted by Prachi Pareekh at 10:14 AM No comments: Links to this post


Labels: modifier error, Passive

SC's Question number - 943, 944

943). With total sales of less than three hundred thousand dollars and fewer new subscribers
than last year, the New England Theatre Company is in danger of losing its building.

(A) of less than three hundred thousand dollars and fewer

(B) lower than three hundred thousand dollars and less

(C) lesser than three hundred thousand dollars and fewer

(D) fewer than three hundred thousand dollars and less

(E) of fewer than three hundred thousand dollars and of fewer

944). Within the boundaries of artistic rivalry lies a sense of family: the shared genetic
inheritance, or accident, that enables musicians to make music.

(A) Within the boundaries of artistic rivalry lies a sense of family

(B) Within the boundaries of artistic rivalry lays a sense of family

(C) A sense of family lies inside of the boundaries of artistic rivalry

(D) A family sense lies within artistic rivalry's boundaries

(E) Within artistic rivalry's boundaries lays a family sense

Answers --

943). OA - A - less vs fewer on the link below

http://gmat-grammar.blogspot.com/2006/08/less-vs-fewer.html

B, D - incorrect - use of less wrong - subscribers are countable so fewer should be used.

C - incorrect - lesser than


E - incorrect - sales are uncountable so fewer is wrong

944). OA - A
Lay = to put something down (Transitive verb - needs direct object) lie = to rest, to recline
(intransitive verb)

B, E - incorrect - use of lay incorrect - sense is not an object so it cannot lay

C - incorrect - inside of the boundaries of - awkward

C, D - incorrect - wrongly modifies the boundaries of artistic rivalry -- should modify a


sense of family.

ShareThisFacebookTweetLinkedInEmailGoogle Pinterest

Posted by Prachi Pareekh at 9:29 AM No comments: Links to this post


Labels: less vs fewer, modifier error

Sunday, March 04, 2007


Gmat Sentence Correction - 99

99). A recent study of ancient clay deposits has provided new evidence supporting the theory of
global forest fires ignited by a meteorite impact that contributed to the extinction of the
dinosaurs and many other creatures some 65 million years ago.

(A) supporting the theory of global forest fires ignited by a meteorite impact that

(B) Supporting the theory that global forest fires ignited by a meteorite impat

(C) that supports the theory of global forest fires that were ignited by a meteorite impact and
that

(D) in support of the theory that global forest fires were ignited by a meteorite impact and that

(E) of support for the theory of a meteorite impact that ignited global forest fires and

Answer --

B is the best choice - theory that - theory that: a belief, a hypothesis Ex: Theory That Life
Began in Volcanic Environment etc

A - incorrect - theory of - theory of: when it is a set of widely accepted facts or principles Ex:
theory of relativity. discrete analytical functions, theory of relativity etc.
C - incorrect - evidence supporting more concise than evidence that supports.(GMAT prefers
concise choices)

D - incorrect - that after and incorrectly introduces a restrictive clause - A recent study of
ancient clay deposits has provided new evidence in support of the theory that global forest
fires were ignited by a meteorite impact conjunction and that contributed to the extinction of
the dinosaurs and many other creatures some 65 million years ago - because how can the
theory contribute to the extinction of...?

E - incorrect - support for the theory

ShareThisFacebookTweetLinkedInEmailGoogle Pinterest

Posted by Prachi Pareekh at 9:01 AM No comments: Links to this post


Labels: wordy

Wednesday, February 28, 2007


SC's Question number - 945, 946

945). Without hearing a word of what is being said or shouted, an


experienced trader on the floor of the stock exchange can listen to the hum
of voices around them and tell what is happening.

(A) Without hearing a word of what is being said or shouted, an


experienced trader

(B) Without hearing a word of what is being said or shouted, experienced


traders

(C) Even though the person has not heard a word of what is being said or
shouted, an experienced trader

(D) Even when the person has not heard a word that is being said or
shouted, experienced traders

(E) In spite of not hearing a word of what is being said or shouted, an


experienced trader

946). Wynton and Branford Marsalis, brothers who have fused the
complex rhythms of contemporary jazz with the rollicking musical legacy
of their hometown, are fitting symbols of the New Orleans jazz revival.
(A) Wynton and Branford Marsalis, brothers who have fused the complex
rhythms of contemporary jazz with the rollicking musical legacy of their
hometown, are fitting symbols of the New Orleans jazz revival.

(B) Wynton and Branford Marsalis are brothers, have fused the complex
rhythms of contemporary jazz with the rollicking musical legacy of their
hometown, and are fitting symbols of the New Orleans jazz revival.

(C) The Marsalis brothers, Wynton and Branford, have fused the complex
rhythms of contemporary jazz with the rollicking musical legacy of their
hometown, which is a fitting symbol of the New Orleans jazz revival.

(D) Fusing the rhythms of contemporary jazz, which are complex, with the
rollicking musical legacy of their hometown, Wynton and Branford
Marsalis are a fitting symbol of the New Orleans jazz revival.

(E) A fitting symbol of the New Orleans jazz revival are brothers Wynton
and Branford Marsalis, who fuse the complex rhythms of contemporary
jazz with the rollicking musical legacy of their hometown.

Answers --

945). OA - B

A, C, E - incorrect - need plural antecedent for they.

D - incorrect - the person...experienced traders

946). A is the best answer.

B - incorrect - awkward

C - incorrect - wrongly suggests that hometown is a fitting symbol of the New Orleans jazz
revival.

D - incorrect - omission of brothers

E - incorrect - subject verb agreement -- A fitting symbol....are brothers....

ShareThisFacebookTweetLinkedInEmailGoogle Pinterest

Posted by Prachi Pareekh at 10:12 PM No comments: Links to this post


Labels: Pronoun error, Subject verb agreement

SC's Question number - 947, 948


947). Young female ballet dancers and gymnasts sometimes fail to maintain good eating habits
caused by the desire to be as thin as possible.

(A) Young female ballet dancers and gymnasts sometimes fail to maintain good eating habits
caused by the desire to be as thin as possible.

(B) Good eating habits sometimes fail to be maintained by young female ballet dancers and
gymnasts caused by desiring to be as thin as possible.

(C) Because they desire to be as thin as possible, good eating habits are sometimes not
maintained by young female ballet dancers and gymnasts.

(D) Because they desire to be as thin as possible, young female ballet dancers and gymnasts
sometimes fail to maintain good eating habits.

(E) Young female ballet dancers and gymnasts sometimes fail to maintain good eating habits
because they desire to be as thin as possible

948). A discussion of our nation's foreign policy must begin with the fact of there being an
independent Western Europe which now thinks of itself in trans-nationalist terms.

(A) A discussion of our nation's foreign policy must begin with the fact of there being

(B) Beginning any discussion of our nation's foreign policy must be the fact of there being

(C) Any discussion of our nation's foreign policy must begin with the fact that there is

(D) Any discussion of our nation's foreign policy must begin by acknowledging the existence
of

(E) To begin discussing our nation's foreign policy there must be an acknowledgment of the
fact that

Answers --

947). OA - D

A - incorrect - illogically suggests - the desire to be as thin as possible causes a failure to


maintain good eating habits

A, B - incorrect - passive

C - incorrect - illogically suggests that good eating habits are the one to desire.
E - incorrect - they ambiguous - refers to eating habits or ballet dancers ?.

NOTE : Because.... is right - clearly acting as a subordinate clause to an independent clause.

948). OA - D - most concise and clear.

A, B - incorrect - use of being

C - incorrect - wordy - with the fact that there is


http://gmat-grammar.blogspot.com/2006/06/few-quick-tips.html

E - incorrect - wordy

ShareThisFacebookTweetLinkedInEmailGoogle Pinterest

Posted by Prachi Pareekh at 10:05 AM No comments: Links to this post


Labels: Passive, Pronoun error, wordy

Sunday, February 25, 2007


SC's Question number - 949, 950

949). Already discussed on the link below - Question number 80.

http://gmatsentencecorrection.blogspot.com/2006/10/gmat-sentence-correction-79-80.html

950). A private house in New York City is a building owned by an individual or individuals
having less than eight units and no commercial space .

(A) a building owned by an individual or individuals having less than eight units and no
commercial space

(B) one that an individual or individuals own with fewer than eight units and no commercial
space

(C) a building with fewer than eight units, no commercial space, and is owned by an
individual or individuals

(D) one that has fewer than eight units, no commercial space and it is owned by an individual
or individuals

(E) one that has fewer than eight units, is owned by an individual or individuals, and has no
commercial space
Answers --

950). E is the best choice - maintains parallelism.

A, B - incorrect - misplaced modifier - units and space incorrectly modifying individuals.

C - incorrect - violating parallelism - missing the verb - has no commercial space

D - incorrect - redundant - use of it not required as subject one is already present.

ShareThisFacebookTweetLinkedInEmailGoogle Pinterest

Posted by Prachi Pareekh at 11:19 PM No comments: Links to this post


Labels: modifier error, PARALLELISM, redundancy

SC's Question number - 951, 952

951). A substance from the licorice plant, 50 times sweeter than sucrose, was recently
discovered, is not only a natural sweetener but also prevents tooth decay.

(A) A substance from the licorice plant, 50 times sweeter than sucrose, was recently discovered

(B) A substance, which was recently discovered, from the licorice plant, 50 times sweeter than
sucrose,

(C) A substance from the licorice plant, which was recently discovered to be 50 times sweeter
than sucrose,

(D) A substance from the licorice plant, 50 times sweeter than sucrose , which was recently
discovered,

(E) A recently discovered substance, 50 times sweeter than sucrose from the licorice plant,

952). After more than four decades of research and development, a new type of jet engine is
being tested that could eventually propel aircraft anywhere in the world within two hours or
help boost cargoes into space at significantly lower costs than current methods permit.

(A) tested that could eventually propel aircraft anywhere in the world within two or help

(B) tested that could eventually have the capability of propelling aircraft anywhere in the
world within two hours or to help

(C) tested, eventually able to propel aircraft anywhere in the world within two hours, or
helping

(D) tested, and it eventually could propel aircraft anywhere in the world within two hours, or
helping

(E) tested, and it could eventually have the capability to propel aircraft anywhere in the world
within two hours or help

Answers --

951). OA given is B. However I do not agree with it.

I pick up C as the best choice - Which in C refers to substance. In general, relative pronouns
usually refer to the immediately preceding words. However exceptions are always there. In my
opinion such a question will not appear on GMAT. Excellent discussion on the usage of
relative pronoun on GMAT club by Paul

A - incorrect - run on sentence

B - incorrect - "50 times sweeter than sucrose" seems to be modifying Licorice plant and not
the substance -"how can a plant be 50 times sweeter than sucrose?".

Also it is wrong to have any kind of parenthetical element right before a prepositional phrase

D - incorrect - which is ambiguous

E - incorrect - suggests wrongly that it was not sucrose which was brought from Licorice
Plant.

952). OA - A.

B - incorrect - violating parallelism - propelling...help

C, D - incorrect - violating parallelism - propel ... helping

E - incorrect - "have the capability to" and "could/can" are redundant. (OG 10th sc no. 87)

ShareThisFacebookTweetLinkedInEmailGoogle Pinterest

Posted by Prachi Pareekh at 9:55 AM No comments: Links to this post


Labels: PARALLELISM, redundancy

SC's Question number - 953, 954

953). Although no proof yet exists of electromagnetic fields generated by household


appliances posing any health threat , mounting scientific evidence has convinced many
experts that there is cause for concern.
(A) of electromagnetic fields generated by household appliances posing any health threat

(B) of electromagnetic fields generated by household appliances that pose any threat to health

(C) that electromagnetic fields generated by household appliances pose any threat to health

(D) that poses any threat to health from electromagnetic fields generated by household
appliances

(E) for any health threat posed by electromagnetic fields generated by household appliances

954). Art historians are using a process known as infrared scanning in analyzing the Mona
Lisa to determine if it has been altered since completion and if Leonardo da Vinci first
sketched the figure in black, as done by many artists of the time.

(A) if it has been altered since completion and if Leonardo da Vinci first sketched the figure in
black, as done

(B) if it had been altered since completion and if Leonardo da Vinci first sketched the figure
in black, a practice employed

(C) whether it has been altered since completion and whether Leonardo da Vinci first sketched
the figure in black, a practice employed

(D) whether it was altered since completion and whether Leonardo da Vinci first sketched the
figure in black, as was done

(E) whether it had been altered since completion and whether Leonardo da Vinci first
sketched the figure in black, a practice done

Answers --

953). OA - C
C can be reconstructed as follows : Although no proof that electromagnetic fields generated by
household appliances pose any threat to health yet exists, mounting scientifc evidence has
convinced many experts that there is cause for concern.
Restrictive Clause : that electromagnetic fields generated by household appliances pose any
threat to health (modifying proof)

A, B - incorrect - "proof of electromagnetic fields" - wrong.

D - incorrect - Proof poses any threat -- wrong


E - incorrect - Passive

954). OA - C - since ---- implies from past till present (now) - so, "has been" is correct

A, B - incorrect - use of "if" - (on GMAT generally whether is preferred over if)

D - incorrect - wrong tense was - must be present tense.

E - incorrect - past perfect (wrong tense) -- illogically suggests alteration was done before the
completion of painting

ShareThisFacebookTweetLinkedInEmailGoogle Pinterest

Posted by Prachi Pareekh at 9:30 AM No comments: Links to this post


Labels: Passive, Tense, Whether vs if

Wednesday, February 21, 2007


SC's Question number - 955, 956

955). Asset allocators create portfolios, often in the form of mutual funds, with the intention to
turn in good results in both "bull" and "bear" markets.

(A) with the intention

(B) the intention of which is

(C) intended

(D) and intending

(E) so intended as

956). Baker was perhaps not the most gifted soloist in the orchestra, but the conductor felt
what was lacking in his technical skill was more than made up by the passion with which he
played the music.

(A) what was lacking in his technical skill was more than made up by

(B) what he lacked in technical skill was more than made up by

(C) whatever was lacking in his technical skill was more than made up by

(D) whatever he lacked in technical skill was more than made up for by

(E) whatever he lacked in technical skill he more than made up by


Answers --

955). C is the best choice. - Asset allocators create portfolios, often in the form of mutual
funds, intended to turn in good results in both "bull" and "bear" markets.

A - incorrect - portfolios cannot have any intention.

B - incorrect - to be successful is not the intention of mutual funds

D - incorrect - Asset allocators create portfolios, often in the form of mutual funds, and
intending to turn in good results in both "bull" and "bear" markets.

E - incorrect - unidiomatic

956). The answer given in 1000 SC's is wrong i.e B. The correct answer is D.
The phrasal verb - to make up for means to compensate for.

ShareThisFacebookTweetLinkedInEmailGoogle Pinterest

Posted by Prachi Pareekh at 11:12 PM No comments: Links to this post


Labels: Idiom

SC's Question number - 957, 958

957). By law, a qualified physician can only prescribe medicine, protecting the public.

(A) By law, a qualified physician can only prescribe medicine, protecting the public.

(B) By law, only a qualified physician can prescribe medicine, protecting the public.

(C) By law, only a qualified physician can prescribe medicine which protects the public.

(D) In order to protect the public, by law a qualified physician only can prescribe medicine.

(E) In order to protect the public, by law only a qualified physician can prescribe medicine.

958). Cartographers have long struggled with the problem of having the spherical Earth to
draw on a flat sheet of paper .

(A) having the spherical Earth to draw on a flat sheet of paper

(B) having a flat sheet of paper on which to draw the spherical Earth

(C) how can one draw the spherical Earth on a flat sheet of paper
(D) how they could use a flat sheet of paper to draw the spherical Earth

(E) how to draw the spherical Earth on a flat sheet of paper

Answers --

957). OA - E

A, B, C - incorrect - awkward - law protect people ..not by law protect people

A, D - incorrect - wrongly says that a qualified physician can only prescribe medicine, and
not do anything else

C - incorrect - wrongly says that medicines are protecting the public

958). E is the best choice -- ....the problem with how to do s-thing.... -- idiomatic

A, B - incorrect - having => possessive - the spherical earth cannot be possessed.

C - incorrect - use of one incorrect

D - incorrect - distorts the meaning - illogically suggests you use paper to draw the spherical
earth.

ShareThisFacebookTweetLinkedInEmailGoogle Pinterest

Posted by Prachi Pareekh at 10:37 PM 1 comment: Links to this post


Labels: Idiom

SC's Question number - 959, 960

959). Child care already a solid part of the employee benefits package at many companies,
more businesses are focusing on a newer family benefit known as elder care, servicing for
older dependents.

(A) Child care

(B) With child care

(C) Child care as

(D) Being the Child care was

(E) With child care's being


960). Constance Horner, chief of the United States government's personnel agency, has
recommended that the use of any dangerous or illegal drug in the five years prior to
application for a job be grounds for not hiring an applicant.

(A) the use of any dangerous or illegal drug in the five years prior to application for a job be
grounds for not hiring an applicant

(B) any dangerous or illegal drug, if used in the five years prior to applying for a job, should
be grounds for not hiring an applicant

(C) an applicant's use of any dangerous or illegal drug in the five years prior to application
for a job be grounds not to hire them

(D) an applicant's use of any dangerous or illegal drug in the five years prior to applying for a
job are grounds that they not be hired

(E) for five years prior to applying for a job, an applicant's use of any dangerous or illegal
drug be grounds for not hiring them

Answers --

959). B is the best choice.

A - incorrect - Child care already a solid part of the employee benefits package at many
companies -> acts as an appositive for more companies ..hence wrongly suggests child care
= more companies.

D, E - incorrect - use of being

C - incorrect - run on sentence

960). A is the best choice.

C, D, E - incorrect - applicant (singular).....them(plural)

B - incorrect - sunjunctive so should must be avoided.

Link below deals with subjunctive verbs.


http://gmat-grammar.blogspot.com/2006_06_01_gmat-grammar_archive.html

ShareThisFacebookTweetLinkedInEmailGoogle Pinterest

Posted by Prachi Pareekh at 10:32 PM No comments: Links to this post


Labels: Passive, Subject verb agreement, subjunctive
Friday, February 16, 2007
SC's Question number - 961, 962

961). Consumer confidence levels, which many economists consider an early indication of the
economy's direction, sagged as the stock market tumbled, but not dramatically enough for
giving a clear picture of new spending patterns.
(A) tumbled, but not dramatically enough for giving

(B) tumbled, but not dramatically enough to give

(C) tumbled, and not so dramatically as to have given

(D) has tumbled, and not dramatically enough to give

(E) has tumbled, but not so dramatically as to give

962). Contrary to the scholarly wisdom of the 1950's and early 1960's that predicted the
processes of modernization and rationalization would gradually undermine it, ethnicity is a
worldwide phenomenon of increasing importance.

(A) would gradually undermine it

(B) to be a gradual undermining of it

(C) would be a gradual undermining of ethnicity

(D) to gradually undermine ethnicity

(E) gradually undermining it

Answers --

961). OA - B

A - incorrect - unidiomatic - for giving

C, D - incorrect - use of and wrong , we need but to show contrast.

D, E - incorrect - use of present perfect tense is wrong here - We need simple past tense here

962). OA - A -- Predicted (future event here is predicted in the past) -- We require would in
this sentence - would is the past tense of will.
"it" refers to ethnicity here.In this particular case the relative pronoun is inversed.
B, D, E - incorrect - use of will incorrect.

C - incorrect - wordy. Also a pronoun should precede the noun ethinicity.

ShareThisFacebookTweetLinkedInEmailGoogle Pinterest

Posted by Prachi Pareekh at 11:04 AM No comments: Links to this post


Labels: Tense, wordy

Thursday, February 15, 2007


SC's Question number - 963, 964

963). For people who have never worked for a living, any job may instill a valuable sense of
self-worth and open doors to better jobs in the future.

(A) may instill a valuable sense of self-worth and open doors to better jobs in the future

(B) might instill for them a valuable sense of self- worth and to open doors to better jobs in the
future

(C) may, in them, instill a valuable sense of self-worth, opening their doors to better jobs in
the future

(D) opening the door later for a better job and giving them a valuable sense of self-worth now

(E) may open the door for a better job later and giving them a valuable sense now of their self-
worth

964). In a leveraged buyout, investors borrow huge sums of money to buy companies, hoping
to pay off the debt by using the company's earnings and to profit richly by the later resale of
the companies or their divisions.

(A) by using the company's earnings and to profit

(B) by using the companies' earnings and by profiting

(C) using the companies' earnings and profiting

(D) with the company's earnings, profiting

(E) with the companies' earnings and to profit

Answers --
963). OA - A

B - incorrect - unidiomatic - instill for

C - incorrect - use of they, them makes this choice awkward

D - incorrect - changes the meaning

E - incorrect - violating parallelism - may open ... giving

964). OA - E

B, C, D - incorrect - violating parallelism - Need to profit to maintain the parallel structure.

A - incorrect - uses company's (singular)

ShareThisFacebookTweetLinkedInEmailGoogle Pinterest

Posted by Prachi Pareekh at 11:51 AM No comments: Links to this post


Labels: Idiom, PARALLELISM

SC's Question number - 965, 966

965). In many upper-class Egyptian homes, French was spoken within the family, just as it
had once been among the Russian aristocracy.

(A) just as it had once been among the Russian aristocracy

(B) just like it once been among the Russian aristocracy

(C) just as Russian aristocracy had once done

(D) similar to what the Russian aristocracy had once done

(E) like what had once been done by the Russian aristocracy

966). In repousse, a method of sculpture, workers lay copper sheets over wooden molds, and
then, using a variety of exotic hammers, carefully pounding the metal into shape.

(A) In repousse, a method of sculpture, workers lay copper sheets over wooden molds, and
then, using a variety of exotic hammers, carefully pounding the metal into shape.

(B) Repousse is method of sculpture in which workers lay copper sheets over wooden molds
and then, using a variety of exotic hammers, carefully pound the metal into shape.

(C) Repousse workers, who do a method of sculpture by laying copper sheets over wooden
molds, use a variety of exotic hammers, and then pounding the metal into shape.

(D) Workers who lay copper sheets over wooden molds use a variety of exotic hammers and
carefully pound the metal into shape in order to do repousse, a method of sculpture.

(E) Laying copper sheets over wooden molds, and using a variety of exotic hammers in order
to pound the metal into shape, repousse is a method of sculpture done by workers

Answers --

965). A is the best choice.

B, E - incorrect - actions are similar hence the use of like is wrong

C, D - incorrect - violating parallelism - first part of the sentence is in passive, so second


half should also be in passive voice.

In many upper-class Egyptian homes, French was spoken within the family, just as it(French)
had once been (spoken)among the Russian aristocracy

966). B is the best choice - correctly emphasises on Repousse and maintains parallelism
pound ...lay

A - incorrect - violating parallelism - pounding is not parallel to lay. Further use of in is


incorrect in In repousse

C, D - incorrect - awkward, change the meaning.

E - incorrect - dangling modifier error - who is laying and using - not stated.

ShareThisFacebookTweetLinkedInEmailGoogle Pinterest

Posted by Prachi Pareekh at 11:09 AM No comments: Links to this post


Labels: modifier error, PARALLELISM

Tuesday, February 13, 2007


SC's Question number - 967, 968

967). In the late nineteenth century Annie Besant was widely regarded as one of the greatest
living public orators, second only to Gladstone in a culture where oratory was the dominant
public medium.

(A) as one of the greatest living public orators, second only


(B) to be one of the greatest living public orators, secondary only

(C) that she was one of the greatest living public orators, secondary only

(D) as being one of the greater living public orators she was only second

(E) to be greater than most other living public orators, and she was second only

968). Interest rates on mortgages have declined steadily during the first six months of this year
but virtually remained unchanged during the next three months.

(A) have declined steadily during the first six months of this year but virtually remained
unchanged

(B) declined steadily during the first six months of this year but virtually remain unchanged

(C) steadily declined during the first six months of this year but remain virtually unchanged

(D) declined steadily during the first six months of this but have remained virtually
unchanging'

(E) declined steadily during the first six months of this year but have remained virtually
unchanged

Answers --

967). A is the best choice. - regard as - correct idiom

B, C, E - incorrect - unidiomatic

D - incorrect - passive .. use of being

968). E is the best choice.

A, B - incorrect - misplaced word modifier - virtually incorrectly modifying remain instead


of unchanged.

C - incorrect - violating verb tense - going from simple past to present tense.

D - incorrect - unchanging is wrong.

ShareThisFacebookTweetLinkedInEmailGoogle Pinterest

Posted by Prachi Pareekh at 12:56 AM No comments: Links to this post


Labels: Idiom, modifier error, Passive, Tense
SC's Question number - 969, 970

969). Leaching, the recovery of copper from the drainage water of mines, as a method of the
extraction of minerals, it was well established as early as the eighteenth century, but until
about 25 years ago miners did not realize that bacteria taken an active part in the process.

(A) as a method of the extraction of minerals, it was well established

(B) as a method of the extraction of minerals well established

(C) was a well-established method of mineral extraction

(D) was a well-established method of extracting mineral that was

(E) had been a method of mineral extraction, well established

970). Lincoln, discovering in young manhood the secret that the Yankee peddler has learned
before him, knew how to use a good story to generate good will.

(A) Lincoln, discovering in young manhood the secret that the Yankee peddler has learned
before him, knew

(B) Discovering in young manhood the secret that the Yankee peddler has learned before him,
Lincoln knew

(C) Lincoln, discovering the secret that the Yankee peddler had learned in young manhood
before him, knew

(D) In young manhood Lincoln discovered the secret that the Yankee peddler had learned
before him:

(E) Lincoln, discovered in young manhood the secret that the Yankee peddler had learned
before him, knew

Answers --

969). OA - C - has correct verb-tense agreement. Correctly makes "well-established" the


adjective of the proper noun "method"

A, B - incorrect - missing verb - verb is necessary here to make first half of the sentence a
clause.

D - incorrect -"that was as early as" - wrongly suggests that the mineral was as early as 18
century
E - incorrect - misplaced modifier - is it the "mineral extraction" or the "method" which is
"well-established"? Further wrongly suggests that leaching is no longer a method of mineral
extraction

970). D is the best among all wrong choices

A, B, C - incorrect - the learning happened before the discovery so past perfect is needed.
C - incorrect - misplaced modifier - "in young manhood before him" wrongly suggests that
there were other manhoods before him

E - incorrect - wrongly suggests that Lincoln was discovered.

NOTE : A present participle is best used to denote an action that is ongoing

ShareThisFacebookTweetLinkedInEmailGoogle Pinterest

Posted by Prachi Pareekh at 12:25 AM No comments: Links to this post


Labels: modifier error, Tense

SC's Question number - 971, 972

971). Many economists predict that the next recession, when it comes, will be caused by
Federal Reserve action taken to prevent an inflationary upsurge that would result if the
economy were to expand at an annual rate of three percent or more.

(A) taken to prevent an inflationary upsurge that would result

(B) they took for preventing an inflationary upsurge that would result

(C) taken to prevent an inflationary upsurge resulting

(D) they took to prevent an inflationary upsurge resulting

(E) taken for preventing an inflationary upsurge that will result

972). Many environmentalists believe that the widespread planting of trees, along with the
conservation of existing forests, would be one of the surest, easiest, and least expensive ways
to begin to halt or even to reverse the buildup of carbon dioxide in the air.

(A) one of the surest, easiest, and least expensive ways to begin to halt or even to reverse

(B) one of the most sure, easy, and least expensive ways to begin a halt or even reverse

(C) one of the surest, easiest, and least expensive ways that would begin halting or even
reversing
(D) a most sure, easy, and inexpensive way beginning the halting and even reversing of

(E) the most sure, easiest, and inexpensive way that would begin halting or even reversing

Answers --

971). OA - A - Conditional - X would result...... if Y were to happen......Subjunctive were ... so


would is correct

B, D - incorrect - they - incorrectly refers to Federal reserve and not to the action.

C - incorrect - resulting wrong

E - incorrect - unidiomatic - taken for --- instead of will would is required.

972). OA - A - some of the surest, easiest least expensive ways to begin to halt

B, E - incorrect - modifier error - "most" modifying sure, easiest, least

C - incorrect - unidiomatic - ways that - must be ways to

D - incorrect - We need definite article the instead of a

ShareThisFacebookTweetLinkedInEmailGoogle Pinterest

Posted by Prachi Pareekh at 12:17 AM No comments: Links to this post


Labels: modifier error, subjunctive

Monday, February 05, 2007


SC's Question number - 973, 974

973). No matter how patiently they explain their reasons for confiscating certain items,
travelers often treat customs inspectors like wanton poachers rather than government
employees .

(A) travelers often treat customs inspectors like wanton poachers rather than government
employees

(B) travelers often treat customs inspectors as wanton poachers instead of government
employees

(C) travelers often treat customs inspectors as if they were not government employees but
wanton poachers
(D) customs inspectors are often treated by travelers as if they were wanton poachers rather
than government employees

(E) customs inspectors are often treated not like government employees but wanton poachers
by travelers

974). One noted economist has made a comparison of the Federal Reserve and an automobile
as racing through a tunnel, bouncing first off one wall, then the other: the car may get where
it is going, but people may be hurt in the process.

(A) made a comparison of the Federal Reserve and an automobile as racing through a tunnel,
bouncing

(B) made a comparison between the Federal Reserve and an automobile racing through a
tunnel, bouncing

(C) compared the federal Reserve with an automobile as racing through a tunnel and which
bounced

(D) compared the Federal Reserve to an automobile racing through a tunnel, bouncing

(E) compared the Federal Reserve with an automobile that races through a tunnel and it
bounces

Answers --

973). Best answer - D

A, B, C - incorrect - Dangling modifiers. -- Link below explains the dangling modifier


concept.
http://gmat-grammar.blogspot.com/2006_05_01_gmat-grammar_archive.html

E - incorrect - violating parallelism - "not... but..." parallelism rule. "not like government
employees,"........."but like wanton poachers."
Also treat like - unidiomatic

NOTE - Pronouns refer not necessarily to the nearest eligible noun at times....there are many
such examples in OG ---
Quasars are so distant that their light has taken billions of years to reach the Earth;
consequently, we see them as they were during the formation of the universe.The pronouns
"them" and "they" refer to "quasars," and not to the nearest eligible noun "years."

In D "they" clearly refers to "customs inspectors." The 1st "they" refers to "customs
inspectors," which is the subject of the sentence.
974). D is the best choice.

"compared to" is used to compare to unlike things to emphasize their similarity.

The economist compared India's economy to a high-speed train. India's economy and the
train are two different entities

A, C - incorrect - " compare as " is unidiomatic.

B - incorrect -"compare between X and Y" is unidomatic.


E - incorrect - referent of it not clear

ShareThisFacebookTweetLinkedInEmailGoogle Pinterest

Posted by Prachi Pareekh at 1:06 AM No comments: Links to this post


Labels: Comparison, Idiom, modifier error, PARALLELISM, Pronoun error

SC's Question number - 975, 976

975). Puritan fanatics brought to civil and military affairs a coolness of judgment and
mutability of purpose that some writers have though inconsistent with their religious zeal, but
which was in fact a natural outgrowth of it.

(A) but which was in fact a natural outgrowth of it

(B) but which were in fact a natural outgrowth of it

(C) but which were in fact natural outgrowths of it

(D) but it was in fact a natural outgrowth of them

(E) which was in fact a natural outgrowth of it

976). Seeking to spur science education, elementary and secondary schools will receive $25
million over the next ten years from the National Science Foundation to promote science.

(A) elementary and secondary schools will receive $25 million over the next ten years from the
National Science Foundation to promote science

(B) $25 million will be spent by the National Science Foundation over the next ten years to
promote science in elementary and secondary schools

(C) over the next ten years $25 million will be spent on elementary and secondary schools by
the National Science Foundation for promoting science
(D) the National Science Foundation is to spend $25 million over the next ten years for
promoting science in elementary and secondary schools

(E) the National Science Foundation will spend $25 million over the next ten years to promote
science in elementary and secondary schools

Answers --

975). Best choice is C -- OA - C - Puritan fanatics brought to civil and military affairs a
coolness of judgment and mutability of purpose that some writers have though inconsistent
with their religious zeal, but which were in fact natural outgrowths of it .

A, D, E - incorrect - was is wrong -- a coolness of judgment and mutability of purpose - need


plural verb were

B - incorrect - outgrowth is incorrect - Coolness of judgement and mutability of purpose are


OUTGROWTHS of their religious zeal.

976). E is the best choice.

A,B, C - incorrect - modifier error - ''National Science Foundation' should modify the clause
" Seeking to spur science education". Hence must be placed close to it.

D - incorrect - is to spend is unidiomatic


- See more at: http://gmatsentencecorrection.blogspot.com/search?updated-max=2007-03-
05T23:33:00-08:00&max-results=20&reverse-paginate=true#sthash.je77hQ0i.dpuf

Das könnte Ihnen auch gefallen